Sunteți pe pagina 1din 46

Share

Report Abuse

Next Blog

Create Blog

Sign In

Blogspot
Top answ ers for Blogspot
www.Answered-Questions.com

Blogspot near Jaipur, 24


Get local answ ers for Blogspot near Jaipur, 24
www.Answered-Questions.com

Searching for Blogspot?


Discover 100+ answ ers for Blogspot
www.Answered-Questions.com Select Chitika |

VISITORS LOCATION

TUESDAY, AUGUST 2, 2011

Indian Freedom Struggle (18571947)


In ancient times, people from all over the world were keen to come to India. The Aryans came from Central Europe and settled down in India.The Persians followed by the Iranians and Parsis immigrated to India. Then came the Moghuls and they too settled down permanently in India. Chengis Khan, the Mongolian, invaded and looted India many times. Alexander the Great too, came to conquer India but went back after a battle with Porus. He-en Tsang from China came in pursuit of knowledge and to visit the ancient Indian universities of Nalanda and Takshila. Columbus wanted to come to India, but instead landed on the shores of America. Vasco da Gama from Portugal came to trade his country's goods in return for Indian species. The French came and established their colonies in India. Lastly, the Britishers came and ruled over India for nearly 200 years. After the battle of Plassey in 1757, the British achieved political power in India. And their paramountcy was established during the tenure of Lord Dalhousie, who became the Governor- General in 1848. He annexed Punjab, Peshawar and the Pathan tribes in the north-west of India. And by 1856, the British conquest and its authority were firmly established. And while the British power gained its heights during the middle of the 19th century, the discontent of the local rulers, the peasantry, the intellectuals, common masses as also of the soldiers who became unemployed due to the disbanding of the armies of various states that were annexed by the British, became widespread. This soon broke out into a revolt which assumed the dimensions of the 1857 Mutiny. THE INDIAN MUTINY OF 1857

Blogspot
Top answ ers for Blogspot w w w .Answ eredQuestions.com

DISASTER MANAGEMENT STUDY MATERIAL

Blogspot near Jaipur, 24


Get local answ ers for Blogspot near Jaipur, 24 w w w .Answ eredQuestions.com

Searching for Blogspot?


Discover 100+ answ ers for Blogspot w w w .Answ eredQuestions.com

Chitika | Select

FIRST TELUGU FREE E-MAGAZINE

CONSTITUTION OF INDIA

The conquest of India, which could be said to have begun with the Battle of Plassey (1757), was practically completed by the end of Dalhousie's tenure in 1856. It had been by no means a smooth affair as the simmering discontent of the people manifested itself in many localized revolt during this period. However, the Mutiny of 1857, which began with a revolt of the military soldiers at Meerut, soon became widespread and posed a grave challenge to the British rule. Even though the British succeeded in crushing it within a year, it was certainly a popular revolt in which the Indian rulers, the masses and the militia participated so enthusiastically that it came to be regarded as the First War of Indian Independence. Introduction of zamindari system by the British, where the peasants were ruined through exorbitant charges made from them by the new class of landlords. The craftsmen were destroyed by the influx of the British manufactured goods. The religion and the caste system which formed the firm foundation of the traditional Indian society was endangered by the British administration. The Indian soldiers as well as people in administration could not rise in hierarchy as the senior jobs were reserved for the Europeans. Thus, there was all-round discontent and disgust against

JULY 2011, VOLUME 07

JUNE 2011, VOLUME 06

the British rule, which burst out in a revolt by the 'sepoys' at Meerut whose religious sentiments were offended when they were given new cartridges greased with cow and pig fat, whose covering had to be stripped out by biting with the mouth before using them in rifles. The Hindu as well as the Muslim soldiers, who refused to use such cartridges, were arrested which resulted in a revolt by their fellow soldiers on May 9, 1857. The rebel forces soon captured Delhi and the revolt spread to a wider area and there was uprising in almost all parts of the country. The most ferocious battles were fought in Delhi, Awadh, Rohilkhand, Bundelkhand, Allahabad, Agra, Meerut and western Bihar. The rebellious forces under the commands of Kanwar Singh in Bihar and Bakht Khan in Delhi gave a stunning blow to the British. In Kanpur, Nana Sahib was proclaimed as the Peshwa and the brave leader Tantya Tope led his troops. Rani Lakshmibai was proclaimed the ruler of Jhansi who led her troops in the heroic battles with the British. The Hindus, the Muslims, the Sikhs and all the other brave sons of India fought shoulder to shoulder to throw out the British. The revolt was controlled by the British within one year, it began from Meerut on 10 May 1857 and ended in Gwalior on 20 June 1858. END OF THE EAST INDIA COMPANY

MAY 2011, VOLUME 05

APRIL 2011, VOLUME 04

Consequent to the failure of the Revolt of 1857 rebellion, one also saw the end of the East India Company's rule in India and many important changes took place in the British Government's policy towards India which sought to strengthen the British rule through winning over the Indian princes, the chiefs and the landlords. Queen Victoria's Proclamation of November 1, 1858 declared that thereafter India would be governed by and in the name of the British Monarch through a Secretary of State. The Governor General was given title of Viceroy, which meant the representative of the Monarch. Queen Victoria assumed the title of the Empress of India and thus gave the British Government unlimited powers to intervene in the internal affair of the Indian states. In brief, the British paramountcy over India, including the Indian States, was firmly established. The British gave their support to the loyal princes, zamindar and local chiefs but neglected the educated people and the common masses. They also promoted the other interests like those of the British merchants, industrialists, planters and civil servants. The people of India, as such, did not have any say in running the government or formulation of its policies. Consequently, people's disgust with the British rule kept mounting, which gave rise to the birth of Indian National Movement.

MARCH 2011, VOLUME - 03

FEBRUARY 2011, VOLUME - 02

The leadership of the freedom movement passed into the hands of reformists like Raja Rammohan Roy, Bankim Chandra and Ishwar Chandra Vidyasagar. During this time, the binding psychological concept of National Unity was also forged in the fire of the struggle against a common foreign oppressor. Raja Rammohan Roy (1772-1833) founded the Brahmo Samaj in 1828 which aimed at purging the society of all its evil practices. He worked for eradicating evils like sati, child marriage and purdah system, championed widow marriage and women's education and favoured English system of education in India. It was through his effort that sati was declared a legal offence by the British.

JANUARY 2011, VOLUME - 01 FOLLOWERS

Swami Vivekananda (1863-1902) the disciple of Ramakrishna Paramahamsa, established the Ramkrishna Mission at Belur

in 1897. He championed the supremacy of Vedantic philosophy. His talk at the Chicago (USA) Conference of World Religions in 1893 made the westerners realize the greatness of Hinduism for the first time. FORMATION OF INDIAN NATIONAL CONGRESS (INC) The foundations of the Indian National Movement were laid by Suredranath Banerjee with the formation of Indian Association at Calcutta in 1876. The aim of the Association was to represent the views of the educated middle class, inspire the Indian community to take the value of united action. The Indian Association was, in a way, the forerunner of the Indian National Congress, which was founded, with the help of A.O. Hume, a retired British official. The birth of Indian National Congress (INC) in 1885 marked the entry of new educated middle-class into politics and transformed the Indian political horizon. The first session of the Indian National Congress was held in Bombay in December 1885 under the president ship of Womesh Chandra Banerjee and was attended among others by and Badr-uddinTyabji.

w ith Google Friend Connect

Members (27) More

Admission into various Institutions in India

Already a member? Sign in

USEFUL SITES CSAT FOR YOU National Cyclone Risk Mitigation Project - The Government of India has sanctioned a Centrally Sponsored Scheme namely National Cyclone Risk Mitigation Project (NCRMP) Phase-I. In this phase State ...
16 hours ago

At the turn of the century, the freedom movement reached out to the common unlettered man through the launching of the "Swadeshi Movement" by leaders such as Bal Gangadhar Tilak and Aurobindo Ghose. The Congress session at Calcutta in 1906, presided by Dadabhai Naoroji, gave a call for attainment of 'Swaraj' a type of self-government elected by the people within the British Dominion, as it prevailed in Canada and Australia, which were also the parts of the British Empire.

SREE MEDIA EDUCATION

BHEL ET

Meanwhile, in 1909, the British Government announced certain reforms in the structure of Government in India which are known as Morley-Minto Reforms. But these reforms came as a disappointment as they did not mark any advance towards the establishment of a representative Government. The provision of special representation of the Muslim was seen as a threat to the Hindu-Muslim unity on which the strength of the National Movement rested. So, these reforms were vehemently opposed by all the leaders, including the Muslim leader Muhammad Ali Jinnah. Subsequently, King George V made two announcements in Delhi: firstly, the partition of Bengal, which had been effected in 1905, was annulled and, secondly, it was announced that the capital of India was to be shifted from Calcutta to Delhi. The disgust with the reforms announced in 1909 led to the intensification of the struggle for Swaraj. While, on one side, the activists led by the great leaders like Bal Gangadhar Tilak, Lala Lajpat Rai and Bipin Chandra Pal waged a virtual war against the British, on the other side, the revolutionaries stepped up their violent activities There was a widespread unrest in the country. To add to the already growing discontent among the people, Rowlatt Act was passed in 1919, which empowered the Government to put people in jail without trial. This caused widespread indignation, led to massive demonstration and hartals, which the Government repressed with brutal measures like the Jaliawalla Bagh massacre, where thousand of unarmed peaceful people were gunned down on the order of General Dyer. JALLIANWALA BAGH MASSACRE Jalianwala Bagh massacre of April 13, 1919 was one of the most inhuman acts of the British rulers in India. The people of Punjab gathered on the auspicious day of Baisakhi at Jalianwala Bagh, adjacent to Golden Temple (Amritsar), to lodge their protest peacefully against persecution by the British Indian Government. General Dyer appeared suddenly with his armed police force and fired indiscriminately at innocent empty handed people

Engineer Trainee Recruitment 2011,BHEL ST Supervisor Trainee Recruitment 2011 - * * Bharat Heavy Electrical Limited BHEL Invites applications for the Recruitment of Engineer Trainees (ET),Supervisor Trainee (ST) Engineering & Finance ...
1 day ago

SOCIO ECONOM Y AT A GLANCE -

UNION PUBLIC SERVICE COMMISSION

PRATIYOGITA DARPAN MAGAZINES FOR DOWNLOAD

Pratiyogita Darpan English FEB 2011 Pratiyogita Darpan English JAN 2011

leaving hundreds of people dead, including women and children. After the First World War (1914-1918), Mohandas Karamchand Gandhi became the undisputed leader of the Congress. During this struggle, Mahatma Gandhi had developed the novel technique of non-violent agitation, which he called 'Satyagraha', loosely translated as 'moral domination'. Gandhi, himself a devout Hindu, also espoused a total moral philosophy of tolerance, brotherhood of all religions, non-violence (ahimsa) and of simple living. With this, new leaders like Jawaharlal Nehru and Subhash Chandra Bose also emerged on the scene and advocated the adoption of complete independence as the goal of the National Movement. THE NON-COOPERATION MOVEMENT

Pratiyogita Darpan English Dec 2010 Pratiyogita Darpan English Nov 2010 Pratiyogita Darpan(English) Oct 2010 PratiyogitaDarpan( English) Sep 2010 Pratiyogita Darpan(English) August 2010 Pratiyogita Darpan(English) July2010 Pratiyogita Darpan (Hindi) July 2010 Pratiyogita Darpan (English) JUNE 2010 Pratiyogita Darpan (Hindi) June 2010 Pratiyogita Darpan (English) May2010 Pratiyogita Darpan (Hindi) May2010 Pratiyogita Darpan(English) March2010 Pratiyogita Darpan(Hindi) April2010 Pratiyogita Darpan(English) Feb2010 Pratiyogita Darpan(English) Jan2010

CATEGORIES A.P. TRIBAL WELFARE EDUCATIONAL SOCIETY (8) APPSC AEE (1) APPSC GROUP-I (22) APPSC GROUP-II (8) CAREER MAGAZINES (11) CIVILS (11) COMMERCE (3) CURRENT AFFAIRS (21) ECONOMY (15) EMPLOYMENT NEWS (2) ENVIRONMENTAL ISSUES (7) EXAM DATES (1) FOREIGN RELATIONS (5) G.K. (2) GENERAL ESSAYS (4) GEOGRAPHY (21) GROUP-I MAINS (9) GROUPS EXAMS. MISCELLANIOUS (2) HALL TICKETS (1) HISTORY (9) INTERNATIONAL ISSUES (2) JOBS (3) LAW (1) LOGOS (2) MISCELLANIOUS (10) MODEL PAPERS (5) NOTIFICATIONS (2) OBJECTIVE QUESTIONS (9) ORGANISATIONS (6) POLITICAL SCIENCE (3) POLITY (26) PREPARATION STRATAGY (7) PREVIOUS QUESTION PAPERS (7) PSC EXAM PAPERS (30) PUBLIC ADMN. (4) RECRUITMENTS (2) REFERENCE BOOKS (1) RESULT (2) SCHEMES (11) SCIENCE AND

The Non-Cooperation Movement was pitched in under leadership of Mahatma Gandhi and the Indian National Congress from September 1920 to February 1922, marking a new awakening in the Indian Independence Movement. After a series of events including the Jallianwala Bagh Massacre, Gandhiji realised that there was no prospect of getting any fair treatment at the hands of British, so he planned to withdraw the nation's co-operation from the British Government, thus launching the NonCooperation Movement and thereby marring the administrative set up of the country. This movement was a great success as it got massive encouragement to millions of Indians. This movement almost shook the British authorities. SIMON COMMISSION

The Non-cooperation movement failed. Therefore there was a lull in political activities. The Simon Commission was sent to India in 1927 by the British Government to suggest further reforms in the structure of Indian Government. The Commission did not include any Indian member and the Government showed no intention of accepting the demand for Swaraj. Therefore, it sparked a wave of protests all over the country and the Congress as well as the Muslim League gave a call to boycott it under the leadership of Lala Lajpat Rai. The crowds were lathi charged and Lala Lajpat Rai, also called Sher-e-Punjab (Lion of Punjab) died of the blows received in an agitation. CIVIL DISOBEDIENCE MOVEMENT

Mahatma Gandhi led the Civil Disobedience Movement that was launched in the Congress Session of December 1929. The aim of this movement was a complete disobedience of the orders of the British Government. During this movement it was decided that India would celebrate 26th January as Independence Day all over the country. On 26th January 1930, meetings were held all over the country and the Congress tricolour was hoisted. The British Government tried to repress the movement and resorted to brutal firing, killing hundreds of people. Thousands were arrested along with Gandhiji and Jawaharlal Nehru. But the movement spread to all the four corners of the country Following this, Round Table Conferences were arranged by the British and Gandhiji attended the second Round Table Conference at London. But nothing came out of the conference and the Civil Disobedience Movement was

APPSC GROUP-1 PRELIMINARY 2010

TECHNOLOGY (10) SOLVED PAPERS (38) SUMMITS (2) TAGLINES (1) UPSC (14)

revived. During this time, Bhagat Singh, Sukhdev and Rajguru were arrested on the charges of throwing a bomb in the Central Assembly Hall (which is now Lok Sabha) in Delhi, to demonstrate against the autocratic alien rule. They were hanged to death on March 23, 1931. QUIT INDIA MOVEMENT

MARKS LIST OF ALL ATTENDED CANDIDATES CLICK HERE

BLOG ARCHIVE 2011 (119) August (2) Indian Freedom Struggle (18571947) Journey of Indian Republic July (30) June (15) May (8) April (12) March (24) February (8) January (20) 2010 (135)

In August 1942, Gandhiji started the 'Quit India Movement' and decided to launch a mass civil disobedience movement 'Do or Die' call to force the British to leave India. The movement was followed, nonetheless, by largescale violence directed at railway stations, telegraph offices, government buildings, and other emblems and institutions of colonial rule. There were widespread acts of sabotage, and the government held Gandhi responsible for these acts of violence, suggesting that they were a deliberate act of Congress policy. However, all the prominent leaders were arrested, the Congress was banned and the police and army were brought out to suppress the movement. Meanwhile, Netaji Subhash Chandra Bose, who stealthily ran away from the British detention in Calcutta, reached foreign lands and organized the Indian National Army (INA) to overthrow the British from India.

APPSC GROUP-I PRELIMINARY EXAM 2010 RESULT CLICK HERE

DOWNLOAD FREE EBOOKS

UPSC QUESTION PAPERS

General Studies
Mains General Studies (Papers I and II) 2003 General Studies (Papers I and II) 2000 General Studies (Papers I and II) 1999 General Studies (Papers I and II) 1998 General Studies (Papers I and II) 1997 Prelims General Studies Paper 2004 with Answers General Studies Paper 2000 General Studies Paper 1998 General Studies Paper 1997

The Second World War broke out in September of 1939 and without consulting the Indian leaders, India was declared a warring state (on behalf of the British) by the Governor General. Subhash Chandra Bose, with the help of Japan, preceded fighting the British forces and not only freed Andaman and Nicobar Islands from the Britishers but also entered the north-eastern border of India. But in 1945 Japan was defeated and Netaji proceeded from Japan through an aeroplane to a place of safety but met with an accident and it was given out that he died in that air-crash itself. "Give me blood and I shall give you freedom" - was one of the most popular statements made by him, where he urges the people of India to join him in his freedom movement. PARTITION OF INDIA AND PAKISTAN At the conclusion of the Second World War, the Labour Party, under Prime Minister Clement Richard Attlee, came to power in Britain. The Labour Party was largely sympathetic towards Indian people for freedom. A Cabinet Mission was sent to India in March 1946, which after a careful study of the Indian political scenario, proposed the formation of an interim Government and convening of a Constituent Assembly comprising members elected by the provincial legislatures and nominees of the Indian states. An interim Government was formed headed by Jawaharlal Nehru. However, the Muslim League refused to participate in the deliberations of the Constituent Assembly and pressed for the separate state for Pakistan. Lord Mountbatten, the Viceroy of India, presented a plan for the division of India into India and Pakistan, and the Indian leaders had no choice but to accept the division, as the Muslim League was adamant. Thus, India became free at the stroke of midnight, on August 14, 1947. (Since then, every year India celebrates its Independence Day on 15th August). Jawaharlal Nehru became the first Prime Minster of free India and continued his term till 1964. Giving voice to the sentiments of the nation, Prime Minister, Pandit Jawaharlal Nehru said, Long years ago we made a tryst with destiny, and now the time comes when we will redeem our pledge, not wholly or in full measure, but very substantially. At the stroke of the midnight hour, when the world sleeps, India will awake to life and freedom. A moment comes, which comes but rarely in history, when we step out from the old to the new, when an age ends and when the soul of a nation, long suppressed, finds utterance.... We end today a period of ill fortune, and India

Public Administration
Mains Public Administration Question Paper of year 2003 Public Administration Question Paper of year 2002 Public Administration Question Paper of year 2000 Public Administration Question Paper of year 1999 Public Administration Question Paper of year 1998 Public Administration Question Paper of year

1997 Prelims Question Paper of year 2003 (Solved) Question Paper of year 2000 Question Paper of year 1999 Question Paper of year 1998 Question Paper of year 1997

discovers herself again. Earlier, a Constituent Assembly was formed in July 1946, to frame the Constitution of India and Dr. Rajendra Prasad was elected its President. The Constitution of India which was adopted by the Constituent Assembly on 26th November 1949. On January 26, 1950, the Constitution was came into force and Dr. Rajendra Prasad was elected the first President of India. You might also like:

Anthropology
Mains Anthropology Question Paper of year 2000 Anthropology Question Paper of year 1999 Anthropology Question Paper of year 1998 Anthropology Question Paper of year 1997 Posted by APPSC PORTAL at 11:29 PM Labels: HISTORY 0 comments
Why India Needs This Pact ?(ASEAN RELATED) Delhi Sultanate ( Medieval India ) UPSC OFFICIAL SAMPLE PAPERS FOR CSAT 2011 PRELIMS Climate change conference in Cancun

LinkWithin

Sociology
Mains Sociology Question Paper of year 2000 Sociology Question Paper of year 1999 Sociology Question Paper of year 1998 Sociology Question Paper of year 1997 Prelims Question Paper of year 2003 (Solved)

Journey of Indian Republic


60 years back, a salute of 21 guns and the unfurling of the Indian National flag by Dr. Rajendra Prasad heralded the historic birth of the Indian Republic on January 26, 1950; 894 days after our country became a dominion following withdrawal of British Rule. Since then, every year the day is celebrated with great pride and happiness all over the nation. The transition of India from a British colony to a sovereign, secular, and democratic nation was indeed historical. It was a long journey of around two decades that started with the conceptualisation of the dream in 1930 to its actual realization in 1950. A look into the journey that led to the birth of Indian Republic will make our celebrations more meaningful. LAHORE SESSION OF THE INDIAN NATIONAL CONGRESS The seeds of a republican nation were sowed at the Lahore session of the Indian National Congress at the midnight of 31st December 1929. The session was held under the presidency of Pt. Jawarhar Lal Nehru. Those present in the meeting took a pledge to mark January 26 as "Independence Day" in order to march towards realizing the dream of complete independence from the British. The Lahore Session paved way to the Civil Disobedience movement. It was decided that January 26, 1930 would be observed as the Purna Swaraj (complete Independence) Day. Many Indian political parties and Indian revolutionaries from all over the country united to observe the day with honour and pride. INDIAN CONSTITUENT ASSEMBLY MEETINGS

Geography
Mains Geography Question Paper of year 2000 Geography Question Paper of year 1999 Geography Question Paper of year 1998 Geography Question Paper of year 1997 Prelims Question Paper of year 2003 (Solved)

Psychology
Mains Psychology Question Paper of year 2000 Psychology Question Paper of year 1999 Psychology Question Paper of year 1998 Psychology Question Paper of year 1997

The Indian Constituent Assembly, which was constituted as a result of the negotiations between the Indian leaders and members of the British Cabinet Mission, had its first meeting on December 9, 1946. The Objective of the Assembly was to give India a constitution, which would serve a lasting purpose and hence appointed a number of committees to thoroughly research the various aspects of the proposed constitution. The recommendations were discussed, debated and revised many times before the Indian Constitution was finalized and officially adopted three years later on November 26, 1949. CONSTITUTION CAME INTO FORCE Though India became a free nation on August 15, 1947, it enjoyed the true spirit of Independence on January 26, 1950 when the Constitution of India finally came into force. The Constitution gave the citizens of India the power to govern themselves by choosing their own government. Dr. Rajendra Prasad, took oath as the first President of India at the Durbar Hall in the

English
Mains

STATE PUBLIC SERVICE COMMISSIONS

English Question Paper of year 2002 English Question Paper of year 2000 English Question Paper of year 1998 English Question Paper of year 1997

Government House and this was followed by the Presidential drive along a five-mile route to the Irwin Stadium, where he unfurled the National Flag. Ever since the historic day, January 26 is celebrated with festivities and patriotic fervor all around the country. The day owes its importance to the constitution of India that was adopted on this day. On this Republic Day, read what the great Constitution of India, that propounds liberal democracy, has in its store. Let's also feel proud in pronouncing what the Preamble to our Constitution says. QUOTES

AP PSC Arunachal Pradesh PSC Assam PSC Bihar PSC Chhattisgarh PSC Goa PSC Gujrat PSC Haryana PSC HP PSC J&K PSC Jharkhand PSC Kerala PSC KPSC Maharashtra PSC Manipur PSC

Essay
Question Paper of year 2001

TOTAL PAGEVIEWS

50,886
Dr. Rajendra Prasad, the first President of Independent India, in his special message to his countrymen, on the birth of the Indian Republic, said: "We must re-dedicate ourselves on this day to the peaceful but sure realization of the dream that had inspired the Father of our Nation and the other captains and soldiers of our freedom struggle, the dream of establishing a classless, co-operative, free and happy society in 'his country'. We must remember that this is more a day of dedications than of rejoicing - dedication to the glorious task of making the peasants and workers the toilers and the thinkers fully free, happy and cultured."

Meghalaya PSC Mizoram PSC MP PSC Nagaland PSC OPSC Punjab PSC Rajasthan PSC Sikkim PSC TN PSC Tripura PSC UK PSC UP PSC West Bengal PSC

COMPETITIVE EXAMS UPSC COMPETITIVE EXAMS Civil Services (I.A.S.) Exam Combined Defence Services (C.D.S.) Exam

C. Rajagopalachari, His Excellency the Governor-General in a broadcast talk from the Delhi Station of All-India Radio on Jan 26, 1950 said: "On the eve of my laying down office, with the inauguration of the Republic, I should like to tender my greetings and best wishes to the men and women of India who will henceforth be a citizen of a republic. I feel deeply thankful for the affection showered on me by all sections of the people, which alone enabled me to bear the burden of an office to the duties and conventions of which I had been an utter stranger."

Combined Medical Services (C.M.S) Exam Engineering Services Exam Geologists Exam Indian Economic/Statistical Services Exam Indian Forest Service (IFS) Exam National Defence Academy (N.D.A.) Exam SSC COMPETITIVE EXAMS

You might also like:

Assistants Grade Exam Clerks Grade Exam Combined Graduate Preliminary Exam Combined Matric Preliminary Exam

UPSC aspirants can now give interview in Indian Languages

National water ways in India

Lok Sabha- An Introduction

UPSC MAINS 2010 GENERAL STUDIES PAPER-1 QUESTION PAPER ...


LinkWithin

Divisional Accountants /Auditors/UDC Exam Income Tax/ Excise Inspectors, etc.. Exam Stenographers' Grade 'C' Exam Stenographers' Grade 'D' Exam DEFENCE COMPETITIVE EXAMS Combined Defence

Posted by APPSC PORTAL at 11:16 PM Labels: POLITY

0 comments

SUNDAY, JULY 31, 2011

Services (C.D.S.) Exam (UPSC) National Defence Academy (N.D.A.) Exam (UPSC) I.A.F. Airman (Technical Trades) Exam I.A.F. Airman (NonTechnical Trades) Exam I.A.F. Airman (Educational Instructors Trade) Exam Indian Navy Sailors Matric Entry Recruitment Exam Indian Navy Artificer Apprentices Exam Indian Navy Dockyard Apprentices Exam Indian Army Soldiers (Technical) M.E.R. Exam Indian Army Soldiers Nursing Assistant's M.E.R. Exam Indian Army Soldiers General Duty (NER) Exam Indian Army Soldiers Clerks Exam L.I.C/ G.I.C COMPETITIVE EXAMS L.I.C Officers' Exam G.I.C Officers' Exam L.I.C Development Officers' Exam G.I.C. Assistants Exam

INDIAN CONSTITUTION
LAW, COURTS AND THE CONSTITUTION India has one of the oldest legal systems in the world. Its law and jurisprudence stretches back into the centuries, forming a living tradition which has grown and evolved with the lives of its diverse people. India's commitment to law is created in the Constitution which constituted India into a Sovereign Democratic Republic, containing a federal system with Parliamentary form of Government in the Union and the States, an independent judiciary, guaranteed Fundamental Rights and Directive Principles of State Policy containing objectives which though not enforceable in law are fundamental to the governance of the nation. SOURCES OF LAW The fountain source of law in India is the Constitution which, in turn, gives due recognition to statutes, case law and customary law consistent with its dispensations. Statutes are enacted by Parliament, State Legislatures and Union Territory Legislatures. There is also a vast body of laws known as subordinate legislation in the form of rules, regulations as well as by-laws made by Central and State Governments and local authorities like Municipal Corporations, Municipalities, Gram Panchayats and other local bodies. This subordinate legislation is made under the authority conferred or delegated either by Parliament or State or Union Territory Legislature concerned. The decisions of the Supreme Court are binding on all Courts within the territory of India. As India is a land of diversities, local customs and conventions which are not against statute, morality, etc. are to a limited extent also recognised and taken into account by Courts while administering justice in certain spheres. ENACTMENT OF LAWS The Indian Parliament is competent to make laws on matters enumerated in the Union List. State Legislatures are competent to make laws on matters enumerated in the State List. While both the Union and the States have power to legislate on matters enumerated in the Concurrent List, only Parliament has power to make laws on matters not included in the State List or the Concurrent List. In the event of repugnancy, laws made by Parliament shall prevail over law made by State Legislatures, to the extent of the repugnancy. The State law shall be void unless it has received the assent of the President, and in such case, shall prevail in that State. APPLICABILITY OF LAWS Laws made by Parliament may extend throughout or in any part of the territory of India and those made by State Legislatures may generally apply only within the terrirory of the State concerned. Hence, variations are likely to exist from State to State in provisions of law relating to matters falling in the State and Concurrent Lists. JUDICIARY One of the unique features of the Indian Constitution is that, notwithstanding the adoption of a federal system and existence of Central Acts and State Acts in their respective spheres, it has generally provided for a single integrated system of Courts to administer both Union and State laws. At the apex of the entire judicial system, exists the Supreme Court of India below which are the High Courts in each State or group of States. Below the High Courts lies a hierarchy of Subordinate Courts. Panchayat Courts also function in some States under various names like Nyaya Panchayat, Panchayat Adalat, Gram Kachheri, etc. to decide civil and criminal disputes of petty and local nature. Different State laws provide for different kinds of jurisdiction of courts. Each State is divided into judicial districts presided over by a District and Sessions Judge, which is the principal civil court of original jurisdiction and can try all offences including those punishable with death. The Sessions Judge is the highest judicial authority in a district. Below him, there are Courts of civil jurisdiction, known in different States as

GRADUATE/PROFESSI ONAL SCHOOLS GRE - Graduate Record Examination GRE Sample Questions GATE - Graduate

Aptitude Test In
Engineering Gate Syllabus SAT - Scholastic Aptitude Test SAT Sample Questions TOEFL - Test Of English as a Foreign Language TOEFL Sample Questions GMAT - Graduate Management Admission Test GMAT Sample Questions

Munsifs, Sub-Judges, Civil Judges and the like. Similarly, the criminal judiciary comprises the Chief Judicial Magistrates and Judicial Magistrates of First and Second Class. CONSTITUTION OF SUPREME COURT On the 28th of January, 1950, two days after India became a Sovereign Democratic Republic, the Supreme Court came into being. The inauguration took place in the Chamber of Princes in the Parliament building which also housed India's Parliament, consisting of the Council of States and the House of the People. It was here, in this Chamber of Princes, that the Federal Court of India had sat for 12 years between 1937 and 1950. This was to be the home of the Supreme Court for years that were to follow until the Supreme Court acquired its own present premises. The inaugural proceedings were simple but impressive. They began at 9.45 a.m. when the Judges of the Federal Court - Chief Justice Harilal J.Kania and Justices Saiyid Fazl Ali, M. Patanjali Sastri, Mehr Chand Mahajan, Bijan Kumar Mukherjea and S.R.Das - took their seats. In attendance were the Chief Justices of the High Courts of Allahabad, Bombay, Madras, Orissa, Assam, Nagpur, Punjab, Saurashtra, Patiala and the East Punjab States Union, Mysore, Hyderabad, Madhya Bharat and Travancore-Cochin. Along with the Attorney General for India, M.C. Setalvad were present the Advocate Generals of Bombay, Madras, Uttar Pradesh, Bihar, East Punjab, Orissa, Mysore, Hyderabad and Madhya Bharat. Present too, were Prime Minister, other Ministers, Ambassadors and diplomatic representatives of foreign States, a large number of Senior and other Advocates of the Court and other distinguished visitors. Taking care to ensure that the Rules of the Supreme Court were published and the names of all the Advocates and agents of the Federal Court were brought on the rolls of the Supreme Court, the inaugural proceedings were over and put under part of the record of the Supreme Court. After its inauguration on January 28, 1950, the Supreme Court commenced its sittings in a part of the Parliament House. The Court moved into the present building in 1958. The building is shaped to project the image of scales of justice. The Central Wing of the building is the Centre Beam of the Scales. In 1979, two New Wings - the East Wing and the West Wing - were added to the complex. In all there are 15 Court Rooms in the various wings of the building. The Chief Justice's Court is the largest of the Courts located in the Centre of the Central Wing. The original Constitution of 1950 envisaged a Supreme Court with a Chief Justice and 7 puisne Judges - leaving it to Parliament to increase this number. In the early years, all the Judges of the Supreme Court sat together to hear the cases presented before them. As the work of the Court increased and arrears of cases began to cumulate, Parliament increased the number of Judges from 8 in 1950 to 11 in 1956, 14 in 1960, 18 in 1978 and 26 in 1986. As the number of the Judges has increased, they sit in smaller Benches of two and three - coming together in larger Benches of 5 and more only when required to do so or to settle a difference of opinion or controversy. The Supreme Court of India comprises the Chief Justice and not more than 25 other Judges appointed by the President of India. Supreme Court Judges retire upon attaining the age of 65 years. In order to be appointed as a Judge of the Supreme Court, a person must be a citizen of India and must have been, for atleast five years, a Judge of a High Court or of two or more such Courts in succession, or an Advocate of a High Court or of two or more such Courts in succession for at least 10 years or he must be, in the opinion of the President, a distinguished jurist. Provisions exist for the appointment of a Judge of a High Court as an Ad-hoc Judge of the Supreme Court and for retired Judges of the Supreme Court or High Courts to sit and act as Judges of that Court. The Constitution seeks to ensure the independence of Supreme Court Judges in various ways. A Judge of the Supreme Court cannot be removed from office except by an order of the President passed after an address in each House of Parliament supported by a majority of the total membership of that House and by a majority of not less than two-thirds of members present and voting, and presented to the President in the same Session for such removal on the ground of proved misbehaviour or incapacity. A person

who has been a Judge of the Supreme Court is debarred from practising in any court of law or before any other authority in India. The proceedings of the Supreme Court are conducted in English only. Supreme Court Rules, 1966 are framed under Article 145 of the Constitution to regulate the practice and procedure of the Supreme Court. SUPREME COURT REGISTRY The Registry of the Supreme Court is headed by the Registrar General who is assisted in his work by three Registrars, four Additional Registrars, twelve Joint Registrars etc. Article 146 of the Constitution deals with the appointments of officers and servants of the Supreme Court Registry. ATTORNEY GENERAL The Attorney General for India is appointed by the President of India under Article 76 of the Constitution and holds office during the pleasure of the President. He must be a person qualified to be appointed as a Judge of the Supreme Court. It is the duty of the Attorney General for India to give advice to the Government of India upon such legal matters and to perform such other duties of legal character as may be referred or assigned to him by the President. In the performance of his duties, he has the right of audience in all Courts in India as well as the right to take part in the proceedings of Parliament without the right to vote. In discharge of his functions, the Attorney General is assisted by a Solicitor General and four Additional Solicitors General. SUPREME COURT ADVOCATES There are three categories of Advocates who are entitled to practise law before the Supreme Court of India:(i) SENIOR ADVOCATES These are Advocates who are designated as Senior Advocates by the Supreme Court of India or by any High Court. The Court can designate any Advocate, with his consent, as Senior Advocate if in its opinion by virtue of his ability, standing at the Bar or special knowledge or experience in law the said Advocate is deserving of such distinction. A Senior Advocate is not entitled to appear without an Advocate-on-Record in the Supreme Court or without a junior in any other court or tribunal in India. He is also not entitled to accept instructions to draw pleadings or affidavits, advise on evidence or do any drafting work of an analogous kind in any court or tribunal in India or undertake conveyancing work of any kind whatsoever but this prohibition shall not extend to settling any such matter as aforesaid in consultation with a junior. (ii) ADVOCATES-ON-RECORD Only these Advocates are entitled to file any matter or document before the Supreme Court. They can also file an appearance or act for a party in the Supreme Court. (iii) OTHER ADVOCATES These are Advocates whose names are entered on the roll of any State Bar Council maintained under the Advocates Act, 1961 and they can appear and argue any matter on behalf of a party in the Supreme Court but they are not entitled to file any document or matter before the Court.
Posted by APPSC PORTAL at 10:28 PM Labels: POLITY 0 comments

FRIDAY, JULY 29, 2011

UPSC CIVIL SERVICES EXAM PAPER-I SOLVED PAPER


1. In the Union Budget 2011-12, a full exemption from the basic customs duty was extended to the bio-based asphalt (bioasphalt). What is the importance of this material?

1. Unlike traditional asphalt, bio-asphalt is not based on fossil fuels. 2. Bioasphalt can be made from non- renewable resources. 3. Bioasphalt can be made from organic waste materials. 4. It is ceo-friendly to use bioasphalt for surfacing of the roads. Which of the statements given above are correct? (a) 1,2 and 3 only (b) 1,3 and 4 only (c) 2 and 4 only (d) 1,2,3 and 4 Ans: b 2. Consider the following:1. Carbon dioxide 2. Oxides of Nitrogen 3. Oxides of Sulphur Which of the above is/are the emission/emissions from coal combustion at thermal power plants? (a) 1 only (b) 2 and 3 only (c) 1 and 3 only (d) 1, 2 and 3 Ans:d 3. Satellites used for telecommunication relay are kept in a geostationary orbit. A satellite is said to be in such an orbit when: 1. The orbit is geosynchronous. 2. The orbit is circular. 3. The orbit lies in the plane of the Earth's equator. 4. The orbit is at an altitude of 22,236 km. Select the correct answer using the codes given below: (a) 1, 2 and 3 only (b) 1, 3 and 4 only (c) 2 and 4 Only (d) 1, 2, 3 and 4 Ans: a 4. India has experienced persistent,and high food inflation in the recent past,what could be the reasons? 1.Due to a gradual switchover to the cultivation of commercial crops, the area under the cultivation of food grains has steadily decreased in the last five years by about 30%. 2. As a consequnce of increasing incomes,the consumption patterns of the people have undergone a significant change. 3. The food supply chain has structural constraints, Which of the statements given above are correct ? (a) 1 and 2 only (b) 2 and 3 only (c) 1 and 3 only (d) 1, 2 and 3 Ans: b 5. At present, scientists can determine the arrangement or relative positions of genes or DNA sequences on a chromosome.How does this knowledge benefit us? 1. It is possible to know the pedigree of livestock. 2. It is possible to understand the causes of all human diseases. 3. It is possible to develop disease-resistant animal breeds, Which of the statements given above is/are correct?

(a) 1 and 2 only (b) 2 Only (c) 1 and 3 only (d) 1, 2 and 3 Ans: c 6. In terms of economy, the visit by foreign nationals to witness the XIX Common Wealth Games in India amounted to (a) Exports (b) Imports (c) Production (d) Consumption Ans: a 7. Microbial fuel cells are considered a source of sustainable energy.Why ? 1. They use living organisms as catalysts to generate electricity from certain substrates. 2. They use a variety of inorganic materials as substrates. 3. They can be installed in waste water treatment plants to cleanse water and produce electricity. Which of the Statements given above is/are correct? (a) 1 only (b) 2 and 3 only (c) 1 and 3 only (d) 1, 2 and 3 Ans: c 8. Which one of the following statements appropriately describes the "fiscal stimulus"? (a) It is a massive investment by the Government in manufacturing sector to ensure the supply of goods to meet the demand surge caused by rapid economic growth. (b) It is an intense affirmati ve action of the Government to boost economic activity in the country (c) It is Government's intensive action on financial institutions to ensure disbursement of loans to agriculture and allied sectors to promote greater food production and contain food inflation (d) It is an extreme affirmative action by the Government to pursue its policy of financial inclusion Ans: b 9. The formation of ozone hole in the Antarctic region has been a cause of concern. What could be the reason for the formation of this hole? (a) Presence of prominent tropo-spheric turbulence; and inflow of chlorofluorocarbons (b) Presence of .prominent polar front and stratospheric': Clouds; and inflow of chlorofluorocarbons (c) Absence of polar front and stratospheric clouds; and inflow of methane and chlorofluorocarbons (d) Increased temperature at polar region due to golbal warming Ans: b 10. Consider the following actions which the Governnient can take: 1. Devaluing the domestic currency. 2. Reduction in the export subsidy. 3. Adopting suitable policies which attract greater FDI and more funds from FIIs. Which of the above action/actions can help in reducing the current account

deficit? (a) 1 and 2 (b) 2 and 3 (c) 3 only (d) 1 and 3 Ans: d 11. The Constitution (Seventy-Third Amend-ment) Act, 1992, which aims at promoting the Panchayati Raj Institutions in the country, provides for which of the following? 1. Constitution of District Planning Committees. 2. State Election Commissions to conduct all panchayat elections. 3. Establishment of State Finance Commissions. Select the correct answer using the codes given below: (a) 1 only (b) 1 ans 2 only (c) 2 and 3 only (d) 1, 2 and 3 Ans: c 12. Two important rivers - one with its source in Jharkhand (and known by a different name in Odisha), and another,with its source in Odisha - merge at a place only a short distance from the coast of Bay of Bengal before flowing into the sea. This is an important site of wildlife and biodiversity and a protected area. Which one of the following could be this? (a) Bhitarkanika (b) Chandipur-on-sea (c) Gopalpur-on-sea (d) Simlipal Ans: a 13. A rapid increase in the rate of inflation is sometimes attributed to the "base effect", What is "base effect" ? (a) It is the impact of drastic deficiency in supply due to failure of crops (b) It is the impact of the - surge in demand due to rapid economic growth (c) It is the impact of the price levels of previous year on the calculation of inflation rate (d) None of the statements (a), (b) and (c) given above is correct in this context Ans: c 14. India is regarded as a country With "Demographic Dividend" This is due to (a) Its high population in the age group below 15 years (b) Its high population in the age group of 15-64 years (c) Its high population in the age group above 65 years (d) Its high total population Ans: b 15. Regarding "carbon credits", which one of the following statements is not correct? (a) The carbon credit system was ratified in conjunction with the Kyoto Protocol (b) Carbon credits are awarded to countries or groups that have reduced greenhouse gases below their emission quota (c) The goal of the carbon credit system is to limit the increase of carbon

dioxide emission (d) Carbon credits are traded at a price fixed from time to time by the United Nations Environment Programme Ans: d 16. Which one of the following is not a feature of "Value Added Tax" ? (a) It is a multi-point destination-based system of taxation (b) It is a tax levied on value addition at each stage of transaction in the production-distribution chain (c) It is a tax on the final consumption of goods or services and must ultimately be borne by the consumer (d) It is basically a subject of the Cental Government and the State Governments are only a facilitator for its successful implementation Ans: d 17. A "closed economy" is an economy in which (a) the money supply is fully controlled (b) deficit financing takes place (c) only exports take place (d) neither exports nor imports take place Ans: d 18. When the bark of a tree is removed in a circular fashion all around near its base,it gradually dries up and dies because (a) Water from soil cannot rise to aerial parts (b) Roots are starved of energy (c) Tree is infected by soil microbes (d) Roots do not receive oxygen for respiration Ans: a 19. The "New START" treaty was In the news. What is this treaty? (a) It is a bilateral strategic nuclear arms reduction treaty between the USA and' the Russian Federation (b) It is a multilateral energy security cooperation treaty among the members of the East Asia Summit (c) It is a treaty between the Russian Federation and the European Union for the energy security cooperation (d) It is a multilateral cooperation treaty among the BRICS countries Species richness Ans: a 20. Three of the following criteria have contributed to the recognition of western Ghats-Sri Lanka and Indo-Burma regions as hotspots of biodiversity: 1. Species richness 2. Vegetation density 3. Endemism. 4. Ethno-botanical importance 5. Threat perception 6. Adaptation of flora and fauna to warm arid humid conditions Which three of the above are correct criteria in this context? (a) 1,2 and 6 (b) 2,4 and 6 (c) 1,3 and 5 (d) 3,4 and 6 Ans: c 21. Human activities in the recent past have caused the increased concentration of carbon dioxide in the atmosphere, but a lot of it does not remain in the lower atmosphere because of 1. Its escape into the outer strato-sphere. 2. The photosynthesis by phyto-plankton in the oceans. 3. The trapping of air in the polar ice caps Which of the statements given above is/are correct? (a) 1 and 2 (b) 2 only (c) 2 and 3 only

(d) 3 only Ans: c 22. In the context of ecosystem productivity, marine upwelling zones are important as they increase the marine productivity by bringing the 1. decomposer microorganisms to the surface. 2. nutrients to the surface. 3. bottom-dwelling organisms to the surface. Which of the statements given above is/are correct ? (a) 1 and 2 (b) 2 only (c) 2 and 3 (d) 3 only Ans: d 23. If a tropical rain forest is removed, it does not regenerate quickly as compared to a tropical deciduous forest. This is because (a) the soil of rain forest is deficient in nutrients (b) propagules of the trees in a rain forest have poor viability (c) the rain forest species are slow- growing (d) exotic species invade the fertile soil of rain forest Ans: a 24. The Himalayan Range is very rich in species diversity. Which one among the following is the most appropriate reason for this Phenomenon? (a) It has a high rainfall that supports luxuriant vegetative growth (b) It is a confluence of different bio-geographical zones (c) Exotic and invasive species have riot been introduced in this region (d) It has less human interference Ans: b 25. With reference to India, consider the following Central Acts: 1. Import and Export (Control) Act, 1947 2. Mining and Mineral Development (Regulation) Act, 1957 3. Customs Act, 1962 4. Indian Forest Act, 1927 Which of the above Acts have relevance to/bearing on the biodiversity conser-vation in the country? (a) 1 and 3 only (b) 2, 3 and 4 only (c) 1, 2, 3 and 4 (d) None of the above Acts Ans: c 26. Karl Marx explained the process of class struggle with the help of which one of the following theories? (a) Empirical liberalism (b) Existentialism (c) Darwin's theory of evolution (d) Dialectical materialism Ans: d 27. A layer in the Earth s atmosphere called Ionosphere facilitates radio communication. Why? 1. The presence of ozone 'causes the,reflection of radio waves to Earth. 2. Radio waves have a very long wavelength Which of the statements given above is/are correct. a) 1 Only b) 2 only c) Both 1 and 2 d) Neither 1 nor 2 Ans: c 28. Both Foreign Direct Investment (FDI) and Foreign Institutional Investor (FII) are related to investment in a country. Which one of the following

statements best represents an important difference between the two ? (a) FII helps bring better management skills and technology, while FDI only brings in capital (b) FII helps in increasing capital availability in general, while FDI only targets specific sectors. (c) FDI flows only into the secondary market, while FII targets primary market (d) FII is considered to be more stable than FDI Ans: b 29. A genetically engineered form of brinjal,Known as the Bt-brinjal, has been developed. The objective of this is (a) To make it pest-resistant (b) To improve its taste and nutritive qualities (c) To make it drought-resistant (d) To make its shelf-life longer Ans: a 30. With reference to "Aam Admi Bima Yojana", consider the following statements: 1. The member insured under the scheme must be the head of the family or an. earning member of the family in a rural landless house- hold. 2. The Member insured must be in the age group of 30 to 65 years. 3. There is a provision for free scholarship for up to two children of the insured who are studying between classes 9 and 12. Which of the statements given above is/ are correct? (a) 1 only (b) 2 and 3 only (c) 1 and 3 only (d) 1, 2 and 3 Ans: c 31. In the context of global oil prices. "Brent crude oil" is frequently referred to in the news. What does this term imply? 1. it is a major classification of crude oil. 2. It is sourced from north sea. 3. It does not contain sulphur. which of the statements given above is/are correct? (a) 2 only (b) 1 and 2 only (c) 1 and 3 only (d) 1, 2 and 3 Ans: b 32. The Function of heavy water in a nuclear reactor is to (a) Slow down the speed of neutrons (b) Increase the speed of neutrons (c) Cool down the reactor (d) Stop the nuclear reaction Ans: a 33. In India, if a religious sect/community is given the status of a national minority, what special advantages it is entitled to ? 1. It can establish and administer exclusive educational institutions. 2. The President of India automatically nominates a representative of the community to Lok Sabha. 3. It can derive benefits from the Prime Minister's 15-Point Programme. Which of the statements given above is/are correct? (a) 1 only (b) 2 and 3 only (c) 1 and 3 onJy (d) 1, 2 and 3 Ans: c

34. India is home to lakhs of persons with disabilities. What are the benefits available to' them under the law? 1. Free schooling till the age of 18 years in government-run schools. 2. Preferential allotment of land for setting up business. 3. Ramps in public buildings. Which of the statements given above is/are correct? (a) 1 only (b) 2 and 3 only (c) 1 and 3 only (d) 1, 2 and 3 Ans: d 35. With what purpose is the Government of India promoting the concept of "Mega Food Parks" ? 1. To provide good infrastructure facilities for the food processing processing industry. 2. To increase the processing of perishable items and reduce wastage. 3. To provide emerging and eco-friendly food processing technologies to entrepreneurs. Select the correct answer using the codes given below: (a) 1 only (b) 1 and 2 only (c) 2 and 3 only (d) 1, 2 and 3 Ans: d 36. The authorization for the withdrawal of funds from the Consolidated Fund of India must come from (a) The President of India (b) The Parliament of India (c) The Prime Minister of India (d) The Union Finance Minister Ans: b 37. All revenues received by the Union Government by way of taxes and other receipts for the conduct of Government business are credited to the (a) Contingency Fund of India (b) Public Account (c) Consolidated Fund of India (d) Deposits and Advances Fund Ans: c 38. Microfinance is the provision of financial services to people of lowincome groups. This includes both the consumers and the selfemployed.The service/ services rendered under micro- finance is/ are : 1. Credit facilities 2. Savings facilities 3. Insurance facilities 4. Fund Transfer facilities Select the correct answer using the codes given below the lists: (a) 1 only (b) 1 and 4 only (C) 2 and 3 only (d) 1, 2, 3 and 4 Ans: d 39. Southeast Asia has captivated the attention. of global community' over space and time as a geostrategically significant region. Which among the following is the most convincing explanation for this global perspective ? (a) It was the hot theatre during the Second World War (b) Its location between the Asian powers of China and India (c) It was the arena of superpower confrontation during the Cold War period (d) Its location between the Pacific and Indian oceans and its pre-eminent

maritime character Ans: d 40. A company marketing food products advertises that its items do not contain transfats. What does this campaign signify to the customers? 1. The food products are not made out of hydrogenated oils. 2. The food products are not made out of animal fats/ oils. 3. The oils used are not likely to damage the cardiovascular health of the consumers. Which of the statements given above is/are correct? (a) 1 only (b) 2 and 3 only (c) 1 and 3 only (d) 1, 2 and 3 Ans: c 41. Among the following who are eligible to benefit from the "Mahatma Gandhi National Rural Employment Guarantee Act" ? (a) Adult members of only the scheduled caste and scheduled tribe households (b) Adult members of below poverty line (BPL) households (c) Adult members of households of all backward communities (d) Adult members of any household Ans: d 42. With reference to "Look East Policy" of India, consider the following statements: 1. India wants to establish itself as an important regional player in the East Asian affairs. 2. India wants to plug the vacuum created by the termination of Cold War. 3. India wants to restore the historical and cultural ties with its neighbours in Southeast and East Asia. Which of the statements given above is/are correct? (a) 1 only (b) 1 and 3 only (c) 3 only (d) 1, 2 and 3 Ans: b 43. When the annual Union Budget is not passed by the Lok Sabha, (a) the Budget is modified and presented again (b) the Budget is referred to the Rajya Sabha for suggestions (c) the Union Finance Minister is asked to resign (d) the Prime Minister submits the resignation of Council of Ministers Ans: d 44. Under the Constitution of India, which one of the following is not a fundamental duty? (a) To vote in public elections (b) To develop the scientific temper (c) To safeguard public property (d) To abide by the Constitution and respect its ideals Ans: a 45. With reference to the Finance Commission of India, which of the following statements is correct? (a) It encourages the inflow of foreign capital for infrastructure development (b) It facilitates the proper distribution of finances among the Public Sector Undertakings (c) It ensuresjransparency in financial administration (d) None of the statements (a), (b) and (c) given above is correct in this context. Ans: d 46. Consider the following: 1. Right to education. 2. Right to equal access to public service. 3. Right to food.

Which of the above is/ are Human Right/Human Rights under "Universal Declaration of Human Rights"? (a) 1 only (b) 1 and 2 only (c) 3 only (d) 1, 2 and 3 Ans: d 47. There is a concern over the increase in harmful algal blooms in the seawaters of India. What could be the causative factors for this phenomenon? 1. Discharge of nutrients from the estuaries. 2. Run-off from the land during the monsoon. 3. Upwelling in the seas. Select the Correct answer from the codes given below: (a) 1 only (b) 1 and 2 only (c) 2 and 3 only (d) 1, 2 and 3 Ans: b 48. Consider the foliowing: 1. Photosynthesis 2. Respiration 3. Decay of organic matter 4. Volcanic action Which of the above add carbon dioxide to the carbon cycle on Earth ? (a) 1 and 4 only (b) 2 and 3 only (c) 2, 3 and 4 only (d) 1, 2,3 and 4 Ans: c 49. Recently, the OSA decided to support India's membership In multilateral export control regimes called the "Australia Group" and the "Wassenaar Arrangement". What is the difference between them? 1. The Australia Group is an informal arrangement which aims to allow exporting countries to minimize the risk of assisting chemical and biological weapons proliferation, whereas the Wassenaar Arrangement is a formal group under the OECD holding identical objectives. 2. The Australia Group comprises predominantly of Asian, African and North American countries, whereas the member countries of Wassenaar'Arrangement are predominantly,from the European Union and American continents. Which of the statements given above is/are correct? (a) 1 only (b) 2 only (c) Both 1 and 2 (d) Neither 1 nor 2 Ans: a 50. The surface of a lake is frozen in severe winter, but the water at its bottom is still liquid. What is the reason? (a) Ice is a bad conductor of heat (b) Since the surface of the lake is at the same temperature as the air, no heat is lost (c) The density of water is maximum at 4C (d) None of the statements (a), (b) and (c) given above is correct. Ans: c

51. A sandy and saline area is the natural habitat of an Indian animal species. The animal has no predators in that area but its existence is threatened due to the destruction of its habitat. Which one of the following could be that animal? (a) Indian wild buffalo (b) Indian wild ass (c) Indian wild boar (d) Indian gazelle Ans: b 52. La Nina is suspected to have caused recent floods in Australia. How IS La Nina different from EI Nino? 1. La Nina is characterised by un-usually cold ocean' temperature in equatorial indian Ocean whereas EI Nino-is -Characterised "by 'unusually warm ocean temperature in the equatorial Pacific Ocean. 2. El Nino has adverse effect on south-west monsoon of India, but La Nina has no effect on monsoon climate. Which of the statements given above is/are correct? (a) 1 only (b) 2 only (c) Both 1 and 2 (d) Neither I nor 2 Ans: d 53. The tendency for increased litigation was visible after the introduction of the land settlement system of Lord Cornwallis in 1793. The reason for this is normally traced to which of the following provisions? (a) Making Zarnindar 's position stronger vis-a-vis the ryot (b) Making East India Company an overlord of Zamindars (c) Making judicial system more efficient (d) None of the (a), (b) and (c) above Ans: b 54. Which one of the following observations is not true about the Quit India Movement of 1942 ? (a) It was a non-violent movement (b) It was led by Mahatma Gandhi (c) It was a spontanous movement (d) It did not attract the labour class in general Ans: b 55. Which amongst the following provided a common factor for tribal insurrection in India in the 19th century ? (a) Introduction of a new system of land revenue and taxation of tribal products (b) Influence of foreign religious missionaries in tribal areas (c) Rise of a large number of money lenders, traders and revenue farmers as middlemen in tribal areas (d) The complete disruption of the old agrarian order of the' tribal communities Ans: c 56. India maintained its early cultural contacts and trade links with Southeast Asia across the Bay of Bengal. For this pre-eminence of early maritime history of Bay of Bengal, which of the following could be the most convincing explanation/explanations? (a) As compared to other countries, India had a better strip-building technology in ancient and medieval times (b) The rulers of southern India always patronized traders, brahmin priests and buddhist monks in this context (c) Monsoon winds across the Bay of Bengal facilitaied.seayo Yagages (d) Both (a) and (b) are convincing explanations inthiscontext Ans: d 57. What' IS the difference between Bluetooth. and Wi-Fi devices? (a) Bluetooth uses 2-4GHz .radio frequency band, whereas Wi-Fi can use

2-4 GHz or 5GHz frequency band (b) Bluetooth is used for Wireless Local Area Networks ,(WLAN) only, whereas Wi-Fi is used for Wireless Wide Area Networks (WWAN) only (c) When information is transmitted between two devices using Blue- tooth technology, the devices have to be in the line of sight of each other, but when Wi-Fi technology is used the devices need not be in the line of sight of each other (d) The statemen (a) and (b) given above are correctin this context Ans: a 58. With reference to micro-irrigation, which of the following statements is/are correct ? 1. Fertilizer/nutrient loss can be reduced. 2. It is the only means of irrigation in dry land farming. 3. In some areas of farming, receding of ground water table can be checked. Select the correct answer using the codes given below: (a) 1 only (b) 2 and 3 only (c) 1 and 3 only (d) 1, 2 and 3 Ans: c 59. With reference to the period of colonial rule in India, "Home -Charges" formed an important part of drain of wealth from India. Which of the following funds constituted "Home Charges" ? 1.Funds used to support the India Office in London. 2. Funds used to pay salaries and pensions of British personnel engaged in India. 3. Funds used for waging wars outside India by the British. Select the correct answer using the codes given below: (a) 1 only (b) I and 2 only (c) 2 and 3 only (d) 1, 2 and 3 Ans: a 60. What was the reason for Mahatma Gandhi to organize a satyagraha on behalf of the peasants of Kheda ? 1. The Administration did not suspend the land revenue collection in spite of a drought. 2. The Administration proposed to introduce Permanent Settlement in Gujarat. Which of the statements given above is/ are correct? (a) 1 only (b) 2 only (c) Both 1 and 2 (d) Neither 1 or 2 Ans: a 61. Biodiversity forms the basis for human existence in the following ways: 1. Soil formation 2. Prevention of soil erosion 3. Recycling of waste 4. Pollination of crops Select the correct answer using the codes given below: (a) 1, 2 and 3 only (b) 2, 3 and 4 only (c) 1 and 4 only (d) 1, 2, 3 and 4 Ans: d 62. Aspartame is an artificial sweetener sold in the market. It consists of amino acids and provides calories like other amino acids. Yet, it is used as a low-calorie sweetening agent in food items. What is the basis of this use ? (a) Aspartame is as sweet as table sugar, but unlike table sugar, it is not

readily oxidized in human body due to lack of requisite enzymes (b) When aspartame is used in food processing, the sweet taste remains, but it 'beccmes resistant to oxida- tion (c) Aspartame is as sweet as sugar, but after ingestion into the body, it is converted into metabolites that yield no calories (d) Aspartame is several times sweeter than table "sugar, hence food items made with small quantities of aspartame yield fewer calorieson oxidation Ans: c 63. What was the purpose with which Sir William Wedderburn and W. S. Caine had set up the Indian Parliamentary Committee in 1893 ? (a) To agitate for Indian political reforms in the House of Commons (b) To campaign for the entry of Indians into the Imperial Judiciary (c) To facilitate a discussion on India's Independence in the British Parliament (d) To agitate for the entry of eminent Indians into the British Parliament Ans: a 64. What is the difference between a CFL and an LED lamp ? 1. To produce light, a CFL uses mercury vapour' and phosphor while an LED lamp uses semi-conductor material. 2. The average life span of a CFL is much longer than that of an LED lamp. 3. A CFL is less energy-efficient as compared to an LED lamp. Which of the statements given above Is/are correct ? (a) 1 only (b) 2 and 3 only (c) 1 and 3 only (d) 1, 2 and 3 Ans: c 65. Recently, "oilzapper" was in the news. What is it ? (a) It is an eco-friendly technology for the remediation of oily sludge and oil spills (b) It is the latest technology developed for under-sea oil exploration (c) It is a genetically engineered high biofuel-yielding maize variety (d) It is the latest technology to control the accidentally caused flames from oil wells Ans: a 66. A married couple adopted a male child. A few years ,later, twin boys were born to them. The blood group of the couple is AB positive and 0 negative. The blood group of. the three sons is A positive, B positive, and 0 positive. The blood group of the adopted son is (a) O positive (b) A positive (c) B positive (d) Cannot be determined on the basis of the given data Ans: a 67. Mahatma Gandhi said that some of his deepest convictions were reflected in a book titled,"Unto this Last" and the book transformed his life. What was the message from the book that transformed Mahatma Gandhi? (a) Uplifting the oppressed and poor is the moral responsibility of an educated man (b) The good of individual is contained in the good of all . (c) The life of celibacy and spiritual pursuit are essential for a noble life (d) All the statements (a), (b) and (c) are correct in this context Ans: 68. With reference to Indian freedom struggle, Usha Mehta is well-known for (a) Running the secret Congress Radio in the wake of Quit India Movement (b) Participating in the Second Round Table Conference (c) Leading a contingent of Indian National Army (d) Assisting in the formation of Interim Government under Pandit Jawaharlal Nehru Ans: a 69. A new optical disc format known as the Blu-ray Disc (BD) is becoming popular. In what way is it different from the traditional DVD?

1. DVD supports Standard Definition video while BD supports High Definition video. 2. Compared to a DVD, the BD format has several times more storage capacity. 3. Thickness of BD is 2-4 mm while that of DVD is1-2 mm. Which of the statements given above is! are correct? (a) 1 only (b) 1 and 2 only (c) 2 and 3 only (d) 1, 2 and 3 Ans: c 70. With reference to the period of Indian freedom struggle, which of the following was/were recommended by the Nehru report ? 1. Complete Independence for India. 2. Joint electorates for reservation of seats for minorities. 3. Provision of fundamental rights for the people of India in the Constitution. Select the correct answer using the codes given below: (a) I only (b) 2 and 3 only (c) 1 and 3 only (d) 1, 2 and 3 Ans: b 71. Among the following States, which one has the most suitable climatic conditions for the cultivation of a large variety of orchids with minimum cost of production, and can develop an export oriented industry in this field? (a) Andhra Pradesh (b) Arunachal Pradesh (c) Madhya Pradesh (d) Uttar Pradesh Ans: b 72. Which one of the following is not a site for in-situ method of conservation of flora? (a) Biosphere Reserve (b) Botanical Garden (c) National Park (d) Wildlife Sanctuary Ans: b 73. Consider the following statements: In India, a Metropolitan Planning Committee? 1. is constituted under the provisions of the Constitution of India. 2. prepares the draft development plans for metropolitan area. 3. has the sole responsibility for implementing Government sponsored schemes III the metropolitan area. Which of the statements given above is/ are correct? (a) 1 and 2 only (b) 2 only (c) 1 and 3 only (d) 1, 2 and 3 Ans: 74. What is the difference between "vote- on-account" and "interim budget" ? 1. The provision of a "vote-on- account" is used by a regular Government, while an "interim budget" is a provision used by a caretaker Government. 2. A "vote-an-account" only deals with the expenditure in Government's budget, while an "interim budget" includes both expenditure and receipts. Which of the statements given above is/ are correct? (a) 1 only (b) 2 only (c) Both 1 and 2 (d) Neither 1 or 2 Ans: 75. Regarding 'the International Monetary Fund, which one of the following

statements, is correct? (a) It can grant loans to any country (b) It can grant loans to only developed countries (c) It grants loans to only member countries (d) It can grant loans to the central bank of a country Ans: c 76. The 2004 Tsunami made people realize that mangroves can serve as a reliable safety hedge against coastal calamities. How do mangroves function as a safety hedge? (a) The mangrove swamps separate the human settlements from the sea by a wide zone in which people neither live nor venture out (b) The mangroves provide both food and medicines which people are in need of after any natural disaster (c) The mangrove trees are tall with dense canopies and serve as an excellent shelter during a cyclone or tsunami (d) The mangrove trees do not get uprooted by storms and tides because of their extensive roots Ans: 77. The Jain philosophy holds that the world is' created 'and maintained by (a) Universal Law (b) Universal Truth (c) Universal Faith (d) Universal Soul Ans: a 78. Salinization occurs when the irrigation water accumulated in the soil evapo- rates, leaving behind salts and minerals. What are the effects of salinization on the irrigated land? (a) It greatly increases the crop production (b) It makes some soils impermeable (c) It raises the water table (d) It fills the air spaces in the soil with water Ans: b 79. The "Red Data Books" published by the International Union for Conservation of Nature and Natural Resources (IUCN) contain lists of 1. Endemic plant and animal species present in the biodiversity hotspots. 2. Threatened plant and animal species. 3. Protected sites for conservation of nature and natural resources . in various countries. Select the correct. answer using the codes given below : (a) 1 and 3 (b) 2 only (c) 2 and 3 (d) 3 only Ans: b 80. Why is the offering of "teaser loans" by commercial banks a cause Of economic concern ? 1. The teaser loans are considered to be an aspect of sub-prime lending and banks may be exposed to the risk of defaulters in future. 2. In India, the teaser loans are mostly given to inexperienced entrepreneurs to set up manufacturing or export units. Which of the statements given above is/ are correct? (a) 1 only (b) 2 only (c) Both 1 and 2 (d) Neither 1 nor 2 Ans: a 81. An artificial satellite orbiting around the Earth does not fall down. This is so because the attraction of Earth (a) does not exist at such distance (b) is neutralized by the attraction of the moon (c) provides the necessary speed for its steady motion

(d) provides the necessary acceleration for its motion Ans: 82. In the context of Indian economy, consider the following statements : 1. The growth rate of GDP has steadily increased in the last five years. 2. The growth rate in per capita income has steadily increased in the last five years. Which of the statements given above is/are correct ? (a) 1 only (b) 2 only (c) Both 1 and 2 (d) Neither 1 nor 2 Ans: 83. In India, which of the following have the highest share in the disbursement of credit to agriculture and allied activities ? (a) Commercial Banks (b) Cooperative Banks (c) Regional Rural Banks (d) Microfinance Institutions Ans: a 84. Which of the following can aid in furthering the Govemment's objective of inclusive growth? 1. Promoting Self-Help Groups 2. Promoting Micro, Small and Medium Enterprises 3. Implementing the Right to Education Act Select the correct answer using the codes given below: (a) 1 only (b) 1 and 2 only (c) 2 and 3 only (d) 1, 2 and 3 Ans: d 85. Why is the Government of India disinvesting its equity in the Central Public Sector Enterprises (CPSEs)? 1. The Government intends to use the revenue earned from the disinvestment mainly to pay back the external debt. 2. The Government no longer intends to retain the management control of the CPSEs. Which of the statements given above is/ are correct? (a) 1 only (b) 2 only (c) Both 1 and 2 (d) Neither 1 nor 2 Ans: 86. What is the difference between asteroids and comets? 1. Asteroids are small rocky plane- toids, while comets are formed of frozen gases held together by rocky and metallic material. 2. Asteroids are found mostly between the _ orbits of Jupiter and Mars, while comets are found mostly between Venus and Mercury. 3. Comets show a perceptible glowing tail, while asteroids do not. Which of the statements given above is/ are correct? (a) 1 and 2 only (b) 1 and 3 only (c) 3 only (d) 1, 2 and 3 Ans: b 87. Economic growth is with usually coupled (a) Deflation (b) Inflation (c) Stagflation (d) Hyperinflation Ans: b

88. The lowering of Bank Rate by the Reserve Bank of India leads to (a) More liquidity in the market (b) Less liquidity in the market (c) No change in the liquidity in the market (d) Mobilization of more deposits by commercial - banks Ans: a 89. Westerlies in southern hemisphere are stronger and persistent than in northern hemisphere. Why? 1. Southern hemisphere landmass as compared hemisphere. has less to northern 2. Coriolis force is higher in southern hemisphere, as compared to northern hemisphere Which of the statements given above isl are correct? (a) 1 only (b) 2 only (c) Both 1 and 2 (d) Neither 1 nor 2 Ans: 90. Between India and East Asia, the navigation time and distance can be greatly reduced by which of the following? 1. Deepening the Malacca straits between Malaysia and Indonesia. 2. Opening a new canal across the Kraisthmus between the Gulf of Siam and Andaman Sea. Which of the statements given above isl are correct? (a) 1 only (b) 2 only (c) Both I and 2 (d) Neither I nor 2 Ans: b 91. Regular intake of fresh fruits and vegetables is recommended in the diet since they are a good source of antioxidants. How do antioxidants help a person maintain health and promote longevity? (a) They activate the enzymes necessary for vitamin synthesis in the body and help prevent vitamin deficiency (b) They prevent excessive oxidation of carbohydrates, fats and proteins in the body and help avoid unnecessary wastage of energy (c) They neutralize the free radicals produced in. the' body during metabolism (d) They activate certain genes in the cells of the body and help delay the ageing process Ans: c 92. Regarding the Indus Valley Civilization, consider the following statements: 1. It was predominantly a secular civilization and the religious element, though present, did not dominate the scene. 2. During this period, cotton was used for manufacturing textiles in India. Which of the statements given above isl are correct? (a) 1 only (b) 2 only (c) Both 1 and 2 (d) Neither 1 nor 2 Ans: c 93. The lower Gangetic plain is characterised by humid climate with high temperature throughout the year. Which one among the following pairs of crops is most suitable for this region? (a) Paddy and cotton (b) Wheat and Jute (c) Paddy and Jute (d) Wheat and cotton Ans: c 94. What could be the main reasonlreasons for the formation of African and Eurasian desert belt ?

1. It is located in the sub-tropical high pressure cells. 2. It is. under the influence of warm ocean currents. Which of the statements given above is/are correct in this context? (a) 1 only (b) 2 only (c) Both 1 and 2 (d) Neither 1 nor 2 Ans: a 95. The jet aircrafts fly very easily and smoothly in the lower stratosphere. What could be the appropriate explanation? 1. There are no clouds or water vapour in the lower stratosphere. 2. There are no vertical winds in the lower stratosphere. which of the statements given above is/are correct in this context? (a) 1 only (b) 2 only (c) Both 1 and 2 (d) Neither 1 nor 2 Ans: a 96. Consider the following statements: 1. Biodiversity is normally greater in the lower latitudes as compared to the higher latitudes. 2. Along the mountain gradients, biodiversity is normally greater in the lower altitudes as compared to the higher altitudes. Which of the statements given above is! are correct? (a) 1 only (b) 2 only (c) Both I and 2 (d) Neither 1 and 2 Ans: c 97. The Brahmaputra, Irrawady and Mekong rivers originate in Tibet and flow it through narrow and parallel mountain ranges in their upper reaches. Of these rivers, Brahmaputra makes a "U" turn in its course to flow into India. This "U" turn is due to (a) Uplift of folded Himalayan series (b) Syntaxial bending of geologically young Himalayas (c) Geo-tectonic disturbance in the tertiary folded mountain chains (d) Both (a) and (b) above Ans: d 98. A state in India has the following characteristics: 1. Its northern part is arid and semiarid. 2. Its central part produces cotton. 3. Cultivation of cash crops is predominant over food crops. Which one of the following states has all of the above characteristics ? (a) Andhra Pradesh (b) Gujarat (c) Karnataka (d) Tamil Nadu Ans: b 99. What is"Virtual Private Network" ? (a) It is a private computer network of an organization where the remote users can transmit encrypted information through the server of the organization (b) It is a computer network across a public internet that provides users access to their organization's network while maintaining the security of the information transmitted (c) It is a computer network in which users can access a shared pool of computing resources through a service provider (d) None of the statements (a), (b) and (c) given above is a correct

description of Virtual Private Network Ans: b 100. The "dharma" and "rita" depict a central idea of ancient Vedic civilization of India. In this context, consider the following statements : 1. Dharma was a conception of obligations and of the discharge of one's duties to oneself and to others. 2. Rita was the fundamental moral law governing the functioning of the universe and all it contained. Which of the statements given above is/ are correct? (a) 1 only (b) 2 only (c) Both 1 and 2 (d) Neither 1 nor 2 Ans: c
Posted by APPSC PORTAL at 9:04 PM Labels: UPSC 0 comments

UPSC CIVIL SERVICES EXAM PAPER-II SOLVED PAPER


Directions for the following 8 (eight) items: Read each of the following two passages and answer the items that follow. Your answers to these items should be based on the passages only. Passage-I For achieving inclusive growth there is-a critical need to rethink the-role of the State. The early debate among economists about the size of the Goverment can be misleading. The need of the hour is to have an enabling Government. India is too large and complex a nation for the State to be able to deliver all that is needed. Asking the Government to produse all the essential goods, create all the necessary jobs, and keep a curb on the prices of all goods is to lead to a large cumbersome bureaucracy and widespread corruption. The aim must be to stay with the objective of inclusive growth that was laid down by the founding fathers of the nation and also to take a more modern view of what the State can realistically deliver. This is what leads to the idea of an enabling State, that is, a Government that does not try to directly deliver to the citizens everything that they need. Instead, it (1) creates an enabling ethos for the market so that individual enterprise can flourish and citizens can, for the most part, provide for the needs of one another, and (2) steps in to help those who do not manage to do well for themselves, for there will always be individuals, no matter what the system, who need support and help. Hence we need a Government that, when it comes to the market, sets effective, incentive-compatible rules and remains on the sidelines with minimal interference, and, at the same time, plays an important role in directly helping the poor by ensuring that they get basic education and health services and receive adequate nutrition and food . 1. According to passage : 1. The objective of inclusive growth was laid down by the founding fathers of the nation. 2. Need of the hour is to have an enabling Government. 3. The Government should engage III maximum interference in market processes. 4. There is a need to change the size of the Government. Which of the statements given above are correct ? (a) 1 and 2 only (b) 2 and 3 only (c) 1 and 4 only (d) 1, 2, 3 and 4 Ans: a 2. According to the passage, the strategy of inclusive growth can be effected by focussing on

(a) meeting all the needs of every citizen in the country. (b) Increasing the regulations over the manufacturing sector. (c) Controlling the distribution of manufacturing goods. (d) Delivery of the basic services to the deprived sections of the society. Ans: d 3. What constitutes an enabling Government? 1. A large bureaucracy. 2. Implementationof welfare programmes through representatives. 3. Creating an ethos that helps individual enterprise 4. Providing resources to those who are underprivileged. 5. Offering direct help to the poor regarding basic services. Select the correct answer from the codes given below : (a) 1, 2 and 3 only (b) 4 and 5 only (c) 3, 4 and 5 only (d) 1, 2, 3, 4 and 5 Ans: c 4. Why is the State unable to deliver "all thai is needed"? 1. It does not. have sufficient bureaucracy. 2. It does not promote inclusive growth. Select the correct answer from the codes given below : (a) 1 only (b) 2 only (c) Both 1 and 2 (d) Neither 1 nor 2 Ans: d 5. What is essential message being conveyed by the author of the passage ? (a) The objectives of inclusive growth laid down by the foundings fathers of the nation should be remembered. (b) The Government needs to make available more schools and health sevices. (c) The Government needs to establish markets and industries to meet the needs of the poor strata of the society. (d) There is a need to rethink the role of the State in achieving inclusive growth. Ans: d Passage-2 The concept of 'creative society' refers to a phase of development of a society in which a large number of potential contradictions become articulate and active. This is most evident when oppressed social groups get politically mobilised and demand their rights. The upsurge of the peasants and tribals, the movements for regional autonomy and selfdetermination, the environmental movements, and the women's movements in the developing countries are signs of emergence of creative society in contemporary times. The forms of social movements and their intensity may vary from country to country and place to place within a country. But the very presence of movements for social transformation in various spheres of a society indicates the emergence of a creative society in a country. 6. What does the author imply by "creative society" ? 1. A society where diverse art forms and literary writings seck incentive. 2. A society where social inequalities are accepted as the norm. 3. A society where a large number of contradictions are recognised. 4. A society where' the exploited and the oppressed groups grow conscious of. their human rights and upliftment. Select the correct answerusing the codes given below : (a) 1, 2 and 3 (b) 4 only (c) 3 and 4 (d) 2 and 4 Ans: c 7. What according to the passage are the manifestations of social movements? 1. Aggressiveness and being incendiary. 2. Instigation by external forces.

3. Quest for social equality and individual freedom. 4. Urge for granting privileges and self-respect to disparaged sections of the society. Select the correct answer using the codes given below: (a) 1 and 3 only (b) 2 and 4 only (c) 3 and 4 only (d) 1, 2, 3 and 4 Ans: c 8. With reference to the passage. consider the following statements: 1. To be a creative society, it is essential to have a variety of social movements. 2. To be a creative society, it is imperative to have potential contradictions and conflicts. Which of the statements given above is/are correct ? (a) 1 only (b) 2 only (c) Both 1 and 2 (d) Neither 1 nor 2 Ans: c 9. Consider the following three statements: 1. Only students can participate in the race. 2. Some participants in the race are girls. 3. All girl participants in the race are invited for coaching. Which one of the following conclusions can be drawn from the above statements? (a) All participants in the race are invited for coaching. (b) All students are invited for coaching. (c) All participants in the race are students. (d) None of the statements (a), (b) and (c) given above is correct. Ans: c Directions for the following 2 (two) items: Each of the following two items consists of four statements. Of these four statements, two cannot both be true, but both can be false. Study the statements carefully and identify the two that satisfy the above condition. Select the correct answer using the codes given below each set of statements : 10. Examine the following statements: 1. All animals are carnivorous. 2. Some animals are not carnivorous. 3. Animals are not carnivorous. 4. Some animals are carnivorous. Codes: (a) 1 and 3 (b) 1 and 2 (c) 2 and 3 (d) 3 and 4 Ans: a 11. Examine the following statements: 1. All trains are run by diesel engine. 2. Some trains are run by diesel engine. 3. No train is run by diesel engine. 4. Some trains are not run by diesel engine. Codes: (a) 1 and 2 (b) 2 and 3 (c) 1 and 3 (d) 1 and 4 Ans: c 12. Consider the four age pyramids given below namely A, B, C and D representing four different countries.

Which one of them indicates the declining population ? (a) A (b) B (c) C (d) D Ans: c 13. The followings figures has four curves namely A, B, C and D, Study the figure and answer the item that follows.

Which curve indicates the exponential growth ? (a) A (b) B (c) C (d) D Ans: c Directions for the following 2 (two) items: The following pie charts show the break-up of disease categories recorded in the patients from two towns, Town A and Town B. Pie charts plot the disease Categories as percentage of the total number of patients. Based on these, answer the two items that follow the charts.

14. Which of the two towns has a higher number of persons with Diabetes? (a) Town A (b) Town B (c) Same in Town A and Town B (d) No inference can be drawn Ans: d 15. What can we say about persons with more than one disease from these graphs ? (a) There are likely to be persons with more than one disease in Town A. (b) There are likely to be persons with more than one disease in Town B. (c) There'are likely to be persons with more than one disease in both Towns A and B. (d) No inference can be drawn. Ans: b 16. Consider the followmg Velocity-Time graph. It shows two trains starting

simultaneously on parallel tracks.

With reference to the above graph, which one of the following statements is not correct ? (a) Train B has an initial acceleration greater than that of Train A. (b) Train B is faster than Train A at all times. (c) Both trains have the same velocity at time to' (d) Both trains travel the same distance in time to units. Ans: d Directions for the following 6 (six) items: Read each of the following two passages and answer the items that follow. Your answers to these items should be based on the passages only. Passage-1 Ecosystems provide people with a variety of goods and services; food, clean water, clean air, flood control, soil stabilization, pollination, climate regulation, spritual fulfilment and aesthetic enjoyment, to name just a few. Most of these benefits either are irreplaceable or the technology necessary to replace them is prohibitively expensive. For example, potable fresh water can be provided by desalinating sea-water, but pnly great cost. The rapidly expanding human population has greatly modified the Earth's ecosystems to meet their increased requirements of some of the goods and services, particularly food, fresh water, timber, fibre and fuel. These modifications have contributed substantially to human well being and economic development. The benefits have not been equally distributed. Some people have actually been harmed by these changes. Moreover, short-term increases in some ecosystem goods and services have come at the cost of the long-term degradation of others. For example, efforts to increase the production of food and fibre have decreased the ability of some ecosystems to provide clean water, regulate flooding and support biodiversity. 17. With reference to the passage, consider the following statements. Expanding human' population has an adverse effect on : 1. Spiritual fulfilment 2. Aesthetic enjoyment 3. Potable fresh water 4. Production of food and fibre 5. Biodiversity Which of the statements given above are correct ? (a) 1, 2 and 3 only (b) 2, 4 and 5 only (c) 3 and 5 only (d) 1, 2, 3, 4 and 5 Ans: c 18. The passage mentions that "some people have actually been harmed by these changes." What does it imply ? 1. The rapid expansion of population has adversely affected some people. 2. Sufficient efforts, have not been made to increase the production of food and fibre. 3. In the short term some people may be harmed, but in the long term everyone will benefit from modifications In the Earth's ecosystems. Which of the statements given above is/are correct? (a) 1 only (b) 2 (c) 1 and 3 (d) None of the statements given above Ans: a 19. With reference to the passage, consider the following statements: 1. It is imperative to modify the Earth's ecosystems for the well being of mankind.

2. Technology can never replace all the goods and services provided by ecosystems. Which of the statements given above is/are correct ? (a) 1 only (b) 2 only (c) Both 1 and 2 (d) Neither 1 nor 2 Ans: b Paasage-B: A moral act must be our own act; must spring from our own will. If we act mechanically, these is no moral content in our act. Such action would be moral, If we think it proper to act like a machine and do so. For, in doing so, we use our discrimination. We should bear in mind the distinction between acting mechanically and acting intentionally. It may be moral of a king to pardon a culprit. But the messenger carrying out the order of pardon plays only a mechanical part in the king's moral act. But if the messenger were to carry out the king's order considering it to be his duty, his action would be a moral one. How can a man understand morality who does not use his own intelligence and power of thought, but lets himself be swept along like a log of wood by a current ? Sometimes a. man defies convention and acts on his own with a view to absolute good. 20. Which of the following statements best describe/describes the thought of the writer? 1. A moral act calls for using our discretion. 2. Man should react to a situation immediately. 3. Man must do his duty. 4. Man should be able to defy convention in order to be moral. Select the correct answer from the codes given below : (a) 1 only (b) 1 and 3 (c) 2 and 3 (d) 1 and 4 Ans: d 21. Which of the following statements is the nearest definition of moral action, according to the writer ? (a) it is a mechanical action based on official orders from superiors. (b) It is an action baased on our sence of discretion. (c) IS a clever action based on the clarity of purpose. (d) It is a religious action based on understanding. Ans: b 22. The passage contains a statement "lets himself be swept along like a log of wood by a current." Among the following statements, which is/are nearest in meaning to this? 1. A person docs not use his own reason. 2. He is susceptible to influence/pressure. 3. He cannot withstand difficulties/challenges. 4. He is like a log of wood. Select the correct answer using the codes given below: (a) 1 only (b) 1 and 2 (c) 2 and 3 (d) 1 and 4 Ans: b 23. Consider the following distance - time graph. The graph shows three athletes A, Band C running side by side for a 30 km race.

With reference to the above graph consider the following statements : 1. the race was won by A. 2. B was ahead of A up to 25 km 26 mark. 3. C ran very slowly from the begining. Which of the statements given above is/are correct ? (a) 1 only (b) 1 and 2 only (c) 2 and 3 only (d) 1, 2 and 3 Ans: b 24. Consider the following figures :

What is the missing number ? (a) 7 (b) 8 (c) 9 (d) 10 Ans: c 25. Study the following figure: A person goes from A to B always moving to the right or downwards along the Jines. How many different routes can he adopt?

Select the correct answer from the codes given below : (a) 4 (b) 5 (c) 6 (d) 7 Ans: c 26. Consider the following figure and answer the item that follows :

What is total number of triangles in the above grid ? (a) 27 (b) 26 (c) 23 (d) 22 Ans: c Directions for the following 4 (four) items: Read the following passage and answer the items that follow. Your answers to these items should be based on the passage only. Passage A country under foreign domination seeks escape from the present in dreams of a vanished age, and finds consolation in visions of past greatness. That is a foolish and dangerous pastime in which many of us indulge. An equally questionable practice for us in India is to imagine that we are still spiritually great though we have come down, in the world in other respects. Spiritual or any other greatness cannot be founded on lack of

freedom and opportunity, or on starvation and misery. Many western writers have encouraged that notion that Indians are other-worldly. I suppose the poor and unfortunate in every country become to some extent other-worldly, unless they become revolutionaries, for this world is evidently not meant for them. So also subject peoples. As a man grows to maturity he is not entirely engrossed in, or satisfied with, the external objective world. He seeks also some inner meaning, some psychological and physical satisfactions. So also with peoples and civilizations as they mature and grow adult. Every civilization and every people exhibit these parallel streams of an external life and an internal life. Where they meet or keep close to each other, there is an equilibrium and stability. When they diverge conflict arises and the crises that torture the mind and spirit. 27. The passage mentions that "this world is evidently not meant for them". It refers to people who 1. seek freedom from foreign domination. 2. live in starvation and misery. 3. become revolutionaries . Which of the statements given above is/are correct ? (a) 1 and 2 (b) 2 only (c) 2 and 3 (d) 3 only Ans: b 28. Consider the following assumptions : 1. A country under foreign domination cannot indulge in spiritual pursuit. 2. Poverty is an impediment in the spiritual pursuit. 3. Subject peoples may become other-wordly. With reference to the passage, which of the above assumptions is/are valid ? (a) 1 and 2 (b) 2 only (c) 2 and 3 (d) 3 only Ans: c 29. The passage thematically centres on (a) the state of mind of oppressed people (b) starvation and misery (c) the growth of civilization (d) body, mind and spirit of people in general Ans: a 30. According to the passage, the torture of the mind and spirit is caused (a) by the impact of foreign domination. (b) by the desire to escape from foreign domination and find consolation III visions of past greatness. (c) due to lack of equilibrium between an external life and an internal life. (d) due to one's inability to be either revolutionary or other-worldly. Ans: c Directions for the following 3 (Three) items: Read the passage given below, study the graph that follows and answer the three items given belwo the figure. During a party, a person was exposed to cotaminated water. A few days later, he developed fever and loose motions. He suffered for some days before going to a doctor for treatment. On starting the treatment, he soon became better and recovered completely a few days later. The following graph shows different phases of the person's disease condition as regions A, B, C, D and E of the curve.

31. Which region/regions of the curve correspond/corresponds to incubation phase of the infection ? (a) A only (b) B only (c) Band C (d) No part of the curve indicates the incubation phase Ans: a 32. Which region of the curve indicates that the person began showing the symptoms of infection ? (a) A (b) B (c) C (d) D Ans: b 33. Which region of the curve indicates that the treatment yielded effective relief? (a) C (b) D (c) E (d) The curve does not indicate the treatment Ans: c 34. There are four routes to travel from city A to city B and six routes from city B to city C. How many routes are possible to travel from the city A to city C ? (a) 24 (b) 12 (c) 10 (d) 8 Ans: a 35. A contract on construction job specifies a penalty for delay in completion of the work beyond a certain date IS as follows : Rs. 200 for the first day, Rs. 250 for the second day, Rs. 300 for the third day etc., the penalty for each succeeding day being 50 more than that of the preceding day. How much penalty should the contractor pay if he delays the work by 10 days ? (a) Rs. 4950 (b) Rs. 4250 (c) Rs. 3600 (d) Rs. 650 Ans: b 36. Consider the figure given below and answer the items that follows:

In the figure shown above, OP 1 and OP 2 are two plane mirrors kept perpendicular to each other. S is the direction of a beam of light falIing on the mirror OP 1. The direction of the reflected beam of light from the mirror OP 2 will be (a) Perpendicular to the direction S. (b) At 45 to the direction S. (c) Opposite and parallel to the direction S. (d) At 60 to the direction S. Ans: c 37. Consider the following tlgure and answer the item that follows:

What is the minimum number of different colours required to paint the figure given above such that no two adjacent regions have the same colour ? (a) 3 (b) 4 (c) 5 (d) 6 Ans: a 38. Consider the following fogure and answer the items that follows:

A square is divided into four rectangles as shown above. The lengths of the sides of rectangles are natural n umbers. The areas of two rectangles are indicated in the figure. What is the length of each side of the square ? (a) 10 (b) 11 (c) 15 (d) Cannot be determined as the given data are Insufficient Ans: b 39. A person has only Rs. 1 and Rs. 2 coins with her. If the total number of coins that she has is 50 and the amount of money with her is Rs. 75, then

the number of Rs. 1 and Rs. 2 coins are, respectively (a) 15 and 35 (b) 35 and 15 (e) 30 and 20 (d) 25 and 25 Ans: d 40. Three persons start walking together and their steps measure 40 cm, 42 cm and 45 cm respectively. What is the minimum distance each should walk so that each can cover the same distance in complete steps ? (a) 25 m 20 cm (b) 50 m 40 cm (c) 75 m 60 cm (d) 100 m 80 cm Ans: a 41. If a bus travels 160 km in 4 hours and a train travels 320 km in 5 hours at uniform speeds, then what is the ratio of the distances travelled by them in one hour ? (a) 8 : 5 (b) 5 : 8 (c) 4 : 5 (d) 1 : 2 Ans: b 42. There are 100 students in a particular class. 60% students play cricket, 30% student play football and 10% student play both the games. What is the number of students who play neither cricket nor football ? (a) 25 (b) 20 (c) 18 (d) 15 Ans: b 43. A village having a population of 4000 requires 150 liters of water per head per day. It has a tank measuring 20 m x 15 m x 6 m. The water of this tank will last for (a) 2 days (b) 3 days (c) 4 days (d) 5 days Ans: b Directions for the following 4 (four) items: Read the follouing passage and answer the items that follow. Your answers to these items should be based on the passage only. Passage A species that exerts. an influence out of proportion to its abundance in an ecosystem is called a keystone species. The keystone species may influence both the species richness of communities and the flow of energy and materials through ecosystems. The sea star Pisaster the flow of energy and materials through ecosystems. The sea star Pisaster ochraceus, which lives in rocky intertidal ecosystems on the Pacific coast of North America, is also an example of a keystone species. Its preferred prey is the mussel Mytilus californianus . In the absence of sea- stars, these mussels crowd out other competitors in a broad belt of the intertidal zone. By consuming mussels, sea star creates bare spaces that are taken over by a variety of other species. A study at the University of washington demonstrated the influence of Pisaster on species richness by removing sea stars from selected parts of the intertidal zone repeatedly over a period of five years. Two major changes occured in the areas from which sea stars were removed. First, the lower edge of the 46. mussel bed extended farther down into the intertidal zone, showing that sea stars are able to eliminate

mussels completely where they are covered with water most of the time. Second, and more dramatically, 28 species of animals and algae disappeared from the sea star removal zone. Eventually only Mytilus, the dominant competitor, occupied the entire substratum. Through its effect on competitive relationships, predation by Pisaster largely determines which species live in these rocky intertidal ecosystems. 44. What is the crux of the passage ? (a) Sea star has a preferred prey. (b) A preferred prey determines the survival of a keystone species. (c) Keystone species ensures species diversity. (d) Sea star is the only keystone species on the Pacific coast of North America. Ans: c 45. With reference to the passage, consider the following statements : 1. Mussels-are generally the dominant species in intertidal ecosystems. 2. The survival of sea stars is generally determined by the abundance of mussels. which of the statements given above is /are correct? (a) 1 only (b) 2 ony (c) Both 1 and 2 (d) Neither 1 nor 2 Ans: d 46. Which of the following is/are implied by the passage ? 1. Mussels are always hard competitors for sea stars. 2. Sea stars of the Pacific coast have reached the climax of their evolution. 3. Sea stars constitute an important component in the energy flow in intertidal ecosystem. Which of the statements given above is/are correct? (a) 1 and 2 (b) 2 only (c) 1 and 3 (d) 3 only Ans: d 47. Consider the following assumptions: 1. The food chains/food web in an influenced ecosystem are keystone species. 2. The presence of keystone species is a specific characteristic of aquatic ecosystems. 3. If the keystone species is completely removed from an ecosystem, it will lead to the collapse of the ecosystem. With reference to the passage, which of the above assumptions is/are valid? (a) 1 only (b) 2 and 3 only (c) 1 nad 3 only (d) 1, 2 and 3 Ans: a 48. Consider the following argument: "In, order to be a teacher one must graduate from college. All poets are poor. Some Mathematicians are poets. No college graduate is poor."

Which one of the following is not a valid conclusion regarding the above argument? (a) Some Mathematicians are not teachers. (b) Some teachers are not Mathematicians. (c) Teachers are not poor. (d) Poets are not teachers. Ans: b 49. A student on her first 3 tests receive. 4 an average score of N points. If she exceeds her previous average score b. 20 points on her fourth test, then what is the average score for the firs 4 tests ?' (a) N + 20 (b) N + 10 (c) N + 4 (d) N + 5 Ans: d 50. In a group of persons, 70% of the persons are male and 30% of the persons are married. If two sevenths of males are married, what fraction of the females is single ? (a) 2/7 (b) 1/3 (c) 3/7 (d) 2/3 Ans: d 51. The houses of A and B face each other on a road going north-south, A's being on the western side. A comes out of his house, turns left, travels 5 km, turns right, travels 5 km to the front of D's house. B does exactly the same and reaches the fron t of C's house. In this context, which one of the following statements is correct ? (a) C and D live on the same street. (b) C's house faces south. (c) The houses of C than 20 km apart. d) None of the above Ans: c Directions for the following 5 (five) items: Read the following passage and answer the items that follow. Your answers to these items should-be based on the passage only. Passage Now India's children have a right to receive at least eight years of education, the gnawing question is whether' it will remain 'on paper' or 'become a reality. One hardly needs a reminder that this right is different from the others enshrined in the Constitution, that the beneficiary - a six year old child cannot demand it, nor can she or hefight a legal battle when the right is denied or violated. In all cases, it is the adult society which must act on behalf of the child. In another peculiarity, where a child's right to education is denied, no compensation offered later can be adequate or relevant. This is so because childhood does not last. if a legal battle fought on behalf of a child is eventually won, it may be of little use to the boy or girl because the opportunity missed at school during childhood cannot serve the same purpose later in life. This may be painfully true for girls because our society permits them only a short childhood, if at all. The Right to Education (RTE) has become law at a point in India's history when the ghastly practice of female infanticide has resurfaced in the form of foeticide. This is "symptomatic of a deeper turmoil" in society which compounding the traditional obstacles to the education of girls. "Tenacious prejudice against the intellectual potential of girls runs across our cultural diversity and the system of education has not been able to address it.

52. With reference to the passage, consider the following statements : 1. When children are denied education, adult society does not act on behalf of them. 2. Right to Education as a law cannot be enforced in the country. Which of the statements given above is/are correct ? (a) 1 only (b) 2 only (c) Both 1 and 2 (d) Neither 1 nor 2 Ans: d 53. According to the passage, what could be 55 the traditional obstacles to the education of girls ? 1. Inability of parents to fight a legal battle when the Right to Education is denied to their children. 2. The traditional way of thinking about girl's role in society. 3. The prejudice against the intellectual potential of girls. 4. Improper system of education. Select the correct answer from the codes given below : (a) 1 and 2 only (b) 2, 3 and 4 only (c) 1, 3 and 4 only (d) 1, 2, 3 and 4 Ans: b 54. On the basis of the passage, consider the following statements: 1. Right to Education is a legal right and not a fundamental right. 2. For realising the goal of universal education, the education system in the country must be made identical to that of developed countries. Which of the statements given above is/are correct ? (a) 1 only (b) 2 only (c) Both 1 and 2 (d) Neither 1 nor 2 Ans: d 55. Which one of the following statements conveys the key message of the passage ? (a) India has declared that education is compulsory for its children. (b) Adult society is not keen on implementing the Right to Education. (c) The Right to Education, particularly of a girl child, needs to be safeguarded. (d) The system of education should be address the issue of right to education. Ans: c 56. Which one of the following statements conveys the inference of the passage ? (a) The society has a tenacious prejudice against the intellectual potential of girls. (b) Adults cannot be relied upon to fight on behalf of children for their Right to Education. (c) The legal fight to get education for children is often protracted and prohibitive. (d) There is no sufficient substitute for education received in childhood. Ans: d

Read the following paesage and answer (three) items that follow: A, B, C, D and E are members of the same family. There are two fathers, two sons, two wives, three males and two females. The teacher was the, wife of a lawyer who was the son of a doctor. E is not male, neither also a wife of a professional. C is the youngest person in the family and D is the eldest. B is a male. 57. How is D realated to E ? (a) Husband (b) Son (c) Father (d) Wife Ans: a 58. Who are the females in the group ? (a) C and E (b) C and D (c) E and A (d) D and E Ans: c 59. Whose wife is the teacher? (a) C (b) D (c) A (d) B Ans: d Read the following passage and ans the 3 (three) items that follow: In a survey regarding a proposal measure to be introduced, 2878 person took part of which 1652 were males. 12 persons voted against the proposal which 796 were males. 1425 persons vote for the proposal. 196 females wet undecided. 60. How many females voted for the proposal ? (a) 430 (b) 600 (c) 624 (d) 640 Ans: b 61. How many males were undecided ? (a) 31 (b) 227 (c) 426 (d) 581 Ans: a 62. How many females were not in favour the proposal ? (a) 430 (b) 496 (c) 586 (d) 1226 Ans: a 63. In a queue, Mr. X is fourteenth from the front and Mr. Y is seventeenth from the end, while Mr. Z is exactly in between Mr. X and Mr. Y. If Mr. X is ahead Mr. Y and there are 48 persons in the queue, how many persons are then between Mr. X and Mr. Z ? (a) 6

(b) 7 (c) 8 (d) 9 Ans: c Directions for the following 9 (nine) items: The following nine items (Questions 64 to 72) are based on three passages in English to test the comprehension of English language and therefore these items do not have Hindi version. Read each passage and answer the items that follow. Passage-I He walked several miles that day but could not get anything to eat or drink except some dry bread and some water, which he got from cottagers and farmers. As night fell, he slept under a haystack lying in a meadow. He felt frightened at first, for the wind blew awfully over the empty fields. He felt cold and hungry, and was feeling more lonely than he had ever felt before. He however, soon fell asleep, being much tired with his long walk. When he got up next day, he was feeling terribly hungry so he purchased a loaf of bread with a few coins that he had. 64. When the night fell, he slept (a) in the open field (b) under a pile of dry grass (c) in a farmer's cottage (d) under a tree Ans: b 65. He soon fell asleep because (a) he was exhausted (b) he was all alone (c) he had not slept for days (d) he was very frightened Ans: a 66. With reference to the passage, consider the following statements : 1. He was walking through the countryside, 2. The cottagers and farmers gave his enough food so that he could sleep at night without feeling hungry. Which of the statements given above is/are correct ? (a) 1 only (b) 2 only (c) Both 1 and 2 (d) Neither 1 nor 2 Ans: a Passage - II I opened the bag and packed the boots in ; and then , just as I was going to close it, a horrible idea occured to meet Had I packed my toothbrush ? I don't know how it is, but I never do know whether I've packed my toothbrush. My toothbrush is a thing that haunts me when I'm travelling, an makes my life a misery, I dream that haven't packed it, and wake up in a col perspiration, and get out of bed and hur for it. And, in the morning, I pack it before I have used it, and it is always the last thing I turn out of the bag; and then repack and forget it, and have to rug upstairs for it at the last moment an carry it to the railway station, wrapped u in my pocket-handkerchief. 67. When he was going to close the bag, the idea that occurred to him was (a) unpleasant (b) sad (c) fantastic (d) amusing

Ans: a 68. What makes his life miserable whenever he undertakes travelling? (a) Going to railway station (b) Forgetting the toothbrush (c) Packing his bag (d) Bad dreams Ans: b 69. His toothbrush is finally (a) in his bag (b) in his bed (c) in his handkerchief (d) lost Ans: c Passage-III

In spring, polar bear mothers emerge from dens with three month old cubs. The mother bear has fasted for as long as eight months but that does not stop the young from demanding full access to her remaining reserves. If there are triplets, the most persistent stands to gain an extra meal and it may have the meal at the expense of others. The smallest of the litter forfeits many meals to stronger siblings. Females are protective of their cubs but tend to ignore family rivalry over food. In 21 years of photographing polar bears, I've only once seen the smallest of triplets survive till autumn. 70. Female polar bears give birth during (a) spnng (b) summer (c) autumn (d) winter Ans: d 71. Mother bear (a) takes sides over cubs (b) lets the cubs fend for themselves (c) feeds only their favourites (d) see that all cubs get an equal share Ans: b 72. With reference to the passage, the following assumptions have been made: 1. Polar bears fast as long as eight months due to non-availability prey. 2. Polar bears always give birth to triplets. Which of the assumptions given above is/are valid? (a) 1 only (b) 2 only (c) Both 1 and 2 (d) Neither 1 nor 2 Ans: d Directions for the following 8 (eight) items: Given below are eight items. Each item describes a situation and is followed by four possible responses. Indicate the response you find most appropriate. Choose only one response for each item. The responses will be evaluated based on the level of appropriatehess for the given situation. Please attempt all the items. There is no penalty for wrong answers

for these eight items. 73. You have been asked to give an explanation for not attending an important official meeting. Your immediate boss who has not informed you about the meeting is now putting pressure on you not to place an allegation against him / her. You would (a) send a written reply explaining the fact. (b) seek an appointment with the top boss to explain the situation. (c) admit your fault to save the situation. (d) put the responsibility on the coordinator of the meeting for not informing. Ans: a 74. A local thug (bad element) has started illegal construction on your vacant plot. He has refused your request to vacate and threatened you of dire consequences in case you do not sell the property at a cheap price to him. You would (a) sell the property at a cheap price to him. (b) go to the police for necessary action. (c) ask for help from your neighbours. (d) negotiate with the goon to get a higher price. Ans: b 75. You have to accomplish a very important task for your headquarters within the next two days. Suddenly you meet with an accident. Your office insists that you complete the task. You would (a) ask for an extension of deadline. (b) inform Headquarters of your inability to finish on time. (c) Suggest alternate person to headquarters who may do the needful. (d) stay away till you recover. Ans: b 76. You are an officer-in-charge for providing basic medical facilities to the survivors of an earthquake affected area. Despite your best possible effort, people put allegations against you for making money out of the funds given for relief. You would (a) let an enquiry be set up to look into the matter. (b) ask your senior to appoint some other person in your place. (c) not pay attention to allegations. (d) stop undertaking any initiative till the matter is resolved. Ans: a 77. You have been made responsible to hire boats at a short notice to be used for an area under flood. On seeing the price mentioned by the boat owners you found that the lowest price was approximately three times more than the approved rate of the Government. You would (a) reject the proposal and call for a fresh price. (b) accept the lowest price. (c) refer the matter to the Government and wait. (d) threaten the boat owners about a possible cancellation of the licence. Ans: b 78. You are the officer-in-charge of a village administering distribution of vaccine in an isolated epidemic hit village, and you are left with only one vaccine. There is a requirement of that vaccine from the GramPradhan and also a poor villager. You are being pressurised by the Gram Pradhan to issue the vaccine to him. You would (a) initiate the procedure to expedite the next supply without issuing the vaccine to either. (b) arrange vaccine for the poor villager from the distributor of another area. (c) ask both to approach a doctor and get an input about the urgency.

(d) arrange vaccine for the Gram Pradhan from the distributor of another area. Ans: c 79. You have taken up a project to create night-shelters for homeless people during the winter season. Within a week of establishing the shelters, you have received complaints from the residents of the area about the increase in theft cases with a demand to remove the shelters. You would (a) ask them to lodge a written complaint in the police station. (b) assure residents of an enquiry into the matter. (c) ask residents to consider the humanitarian effort made. (d) continue with the project and ignore their complaint. Ans: b 80. You, as an administrative authority, have been approached, by the daughter-in-law of an influential person regarding harassment by her in-laws on account of insufficient dowry. Her parents are not able to approach you because of social pressures. You would (a) call the in-laws for an explanation. (b) counsel the lady to adjust, given such a circumstance. (c) take action after her parents approach you. (d) ask her to lodge complaint with the police. Ans: d
Posted by APPSC PORTAL at 9:03 PM Labels: UPSC 0 comments

Home
Subscribe to: Posts (Atom)
Tag Cloud

Older Posts

Picture Window template. Powered by Blogger.

S-ar putea să vă placă și